Download as pdf or txt
Download as pdf or txt
You are on page 1of 126

1

PONDICHERRY
UNIVERSITY MBBS
CURRICULUM 2019

CHAPTER I

GENERAL CONSIDERATIONS AND TEACHING


APPROACH

1. Introduction

2. Objectives of the Indian Medical Graduate Training Programme


2.1. National Goals
2.2. Institutional Goals
2.3. Goals and Roles for the Learner

3. Competency Based Training Programme of the Indian Medical


Graduate

4. Broad Outline on training format


2

CHAPTER II

ADMISSION TO INDIAN MEDICAL GRADUATE


PROGRAMME: NATIONAL ELIGIBILITY-CUM-
ENTRANCE TEST AND COMMON COUNSELLING
5. Admission to the Indian Medical Graduate Programme

CHAPTER III

6. Migration

CHAPTER IV

PHASE WISE TRAINING AND TIME


DISTRIBUTION FOR PROFESSIONAL
DEVELOPMENT
7. Training period and time distribution:
8. Phase distribution and timing of examination
9. New teaching / learning elements
9.1. Foundation Course
9.2. Early Clinical Exposure
9.3. Electives
9.4.Professional Development including Attitude, Ethics
and Communication Module (AETCOM)
9.5. Learner-doctor method of clinical training (Clinical Clerkship)
3

CHAPTER V
COMPETENCY BASED
CURRICULUM OF
THE INDIAN MEDICAL GRADUATE PROGRAMME

10. Specific Competencies


10.1. Preamble
10.2. Integration
10.3. Pre-clinical Subjects
10.4. Second Professional (Para-Clinical)
10.5. Third Professional (Part I)
10.6. Third Professional (Part II)

CHAPTER VI
ASSESSMENT

11. Assessment

11.1. Eligibility to appear for Professional examinations


11.2. University Examinations

CHAPTER VII
INTERNSHIP
12. Internship
12.1. Goal
12.2. Objectives
12.3. Time
Distribution
12.4. Other details

12.5. Assessment of Internship


12.6. Internship – discipline related
4

CHAPTER I
GENERAL CONSIDERATIONS AND TEACHING APPROACH
1. The provisions contained in Part II of these Regulations shall apply to the MBBS
course starting from academic year 2019-20 onwards
2. Indian Medical Graduate Training Programme
The undergraduate medical education programme is designed with a goal to create an
“Indian Medical Graduate” (IMG) possessing requisite knowledge, skills, attitudes,
values and responsiveness, so that she or he may function appropriately and effectively
as a physician of first contact of the community while being globally relevant. To
achieve this, the following national and institutional goals for the learner of the Indian
Medical Graduate training programme are hereby prescribed:-
2.1. National Goals
At the end of undergraduate program, the Indian Medical Graduate should be able to:
(a) Recognize “health for all” as a national goal and health right of all citizens and
by undergoing training for medical profession to fulfill his/her social
obligations towards realization of this goal.
(b) Learn every aspect of National policies on health and devote her/him to its
practical implementation.
(c) Achieve competence in practice of holistic medicine, encompassing promotive,
preventive, curative and rehabilitative aspects of common diseases.
(d) Develop scientific temper, acquire educational experience for proficiency in
profession and promote healthy living.
(e) Become exemplary citizen by observance of medical ethics and fulfilling social
and professional obligations, so as to respond to national aspirations.
2.2. Institutional Goals
(1) In consonance with the national goals each medical institution should evolve
institutional goals to define the kind of trained manpower (or professionals)
they intend to produce. The Indian Medical Graduates coming out of a medical
institute should:
(a) be competent in diagnosis and management of common health problems of the
individual and the community, commensurate with his/her position as a
member of the health team at the primary, secondary or tertiary levels, using
his/her clinical skills based on history, physical examination and relevant
investigations.
(b) be competent to practice preventive, promotive, curative, palliative and
rehabilitative medicine in respect to the commonly encountered health
problems.
(c) appreciate rationale for different therapeutic modalities; be familiar with the
administration of “essential medicines” and their common adverse effects.
5

(d) be able to appreciate the socio-psychological, cultural, economic and


environmental factors affecting health and develop humane attitude towards
the patients in discharging one's professional responsibilities.
(e) possess the attitude for continued self learning and to seek further expertise
or to pursue research in any chosen area of medicine, action research and
documentation skills.
(f) be familiar with the basic factors which are essential for the implementation
of the National Health Programmes including practical aspects of the
following:
(i) Family Welfare and Maternal and Child Health (MCH)
(ii) Sanitation and water supply
(iii) Prevention and control of communicable and non-communicable
diseases
(iv) Immunization
(v) Health Education
(vi) Indian Public Health Standards (IPHS), at various levels of service
delivery
(vii) Bio-medical waste disposal
(viii) Organizational and/or institutional arrangements.
(g) acquire basic management skills in the area of human resources, materials
and resource management related to health care delivery, hospital
management, inventory skills and counseling.
(h) be able to identify community health problems and learn to work to resolve
these by designing, instituting corrective steps and evaluating outcome of such
measures.
(i) be able to work as a leading partner in health care teams and acquire
proficiency in communication skills.
(j) be competent to work in a variety of health care settings.
(k) have personal characteristics and attitudes required for professional life such
as personal integrity, sense of responsibility and dependability and ability to
relate to or show concern for other individuals.
(2) All efforts must be made to equip the medical graduate to acquire the skills as
detailed in Table 11 Certifiable procedural skills – A Comprehensive list of
skills recommended as desirable for Bachelor of Medicine and Bachelor of
Surgery (MBBS) – Indian Medical Graduate.
2.3. Goals and Roles for the Learner
In order to fulfil the goal of the IMG training programme, the medical graduate
must be able to function in the following roles appropriately and effectively:-
2.3.1. Clinician who understands and provides preventive, promotive, curative,
palliative and holistic care with compassion.
2.3.2. Leader and member of the health care team and system with capabilities to
collect analyze, synthesize and communicate health data appropriately.
6

2.3.3. Communicator with patients, families, colleagues and community.


2.3.4. Lifelong learner committed to continuous improvement of skills and
knowledge.
2.3.5. Professional, who is committed to excellence, is ethical, responsive and
accountable to patients, community and profession.
3. Competency Based Training Programme of the Indian Medical Graduate
Competency based learning would include designing and implementing medical
education curriculum that focuses on the desired and observable ability in real life
situations. In order to effectively fulfil the roles as listed in clause 2, the Indian Medical
Graduate would have obtained the following set of competencies at the time of
graduation:
Clinician, who understands and provides preventive, promotive, curative,
palliative and holistic care with compassion
3.1.1. Demonstrate knowledge of normal human structure, function and development
from a molecular, cellular, biologic, clinical, behavioural and social perspective.
3.1.2. Demonstrate knowledge of abnormal human structure, function and
development from a molecular, cellular, biological, clinical, behavioural and
social perspective.
3.1.3. Demonstrate knowledge of medico-legal, societal, ethical and humanitarian
principles that influence health care.
3.1.4. Demonstrate knowledge of national and regional health care policies including
the National Health Mission that incorporates National Rural Health Mission
(NRHM) and National Urban Health Mission (NUHM), frameworks, economics
and systems that influence health promotion, health care delivery, disease
prevention, effectiveness, responsiveness, quality and patient safety.
3.1.5. Demonstrate ability to elicit and record from the patient, and other relevant
sources including relatives and caregivers, a history that is complete and
relevant to disease identification, disease prevention and health promotion.
3.1.6. Demonstrate ability to elicit and record from the patient, and other relevant
sources including relatives and caregivers, a history that is contextual to
gender, age, vulnerability, social and economic status, patient preferences,
beliefs and values.
3.1.7. Demonstrate ability to perform a physical examination that is complete and
relevant to disease identification, disease prevention and health promotion.
3.1.8. Demonstrate ability to perform a physical examination that is contextual to
gender, social and economic status, patient preferences and values.
3.1.9. Demonstrate effective clinical problem solving, judgment and ability to
interpret and integrate available data in order to address patient problems,
generate differential diagnoses and develop individualized management plans
that include preventive, promotive and therapeutic goals.
3.1.10. Maintain accurate, clear and appropriate record of the patient in conformation
with legal and administrative frame works.
7

3.1.11. Demonstrate ability to choose the appropriate diagnostic tests and interpret
these tests based on scientific validity, cost effectiveness and clinical context.
3.1.12. Demonstrate ability to prescribe and safely administer appropriate therapies
including nutritional interventions, pharmacotherapy and interventions based
on the principles of rational drug therapy, scientific validity, evidence and cost
that conform to established national and regional health programmes and
policies for the following:
(i) Disease prevention,
(ii) Health promotion and cure,
(iii) Pain and distress alleviation, and
(iv) Rehabilitation.
3.1.13. Demonstrate ability to provide a continuum of care at the primary and/or
secondary level that addresses chronicity, mental and physical disability.
3.1.14. Demonstrate ability to appropriately identify and refer patients who may
require specialized or advanced tertiary care.
3.1.15. Demonstrate familiarity with basic, clinical and translational research as it
applies to the care of the patient.
3.2. Leader and member of the health care team and system
3.2.1. Work effectively and appropriately with colleagues in an inter-professional
health care team respecting diversity of roles, responsibilities and
competencies of other professionals.
3.2.2. Recognize and function effectively, responsibly and appropriately as a health
care team leader in primary and secondary health care settings.
3.2.3. Educate and motivate other members of the team and work in a collaborative
and collegial fashion that will help maximize the health care delivery potential
of the team.
3.2.4. Access and utilize components of the health care system and health delivery in
a manner that is appropriate, cost effective, fair and in compliance with the
national health care priorities and policies, as well as be able to collect, analyze
and utilize health data.
3.2.5. Participate appropriately and effectively in measures that will advance quality
of health care and patient safety within the health care system.
3.2.6. Recognize and advocate health promotion, disease prevention and health care
quality improvement through prevention and early recognition: in a) life style
diseases and b) cancers, in collaboration with other members of the health care
team.
3.2.7. Communicator with patients, families, colleagues and community
Demonstrate ability to communicate adequately, sensitively, effectively and
respectfully with patients in a language that the patient understands and in a
manner that will improve patient satisfaction and health care outcomes.
8

Demonstrate ability to establish professional relationships with patients and


families that are positive, understanding, humane, ethical, empathetic, and
trustworthy.
Demonstrate ability to communicate with patients in a manner respectful of
patient’s preferences, values, prior experience, beliefs, confidentiality and
privacy.
Demonstrate ability to communicate with patients, colleagues and families in a
manner that encourages participation and shared decision-making.
3.3. Lifelong learner committed to continuous improvement of skills and
knowledge
3.3.1. Demonstrate ability to perform an objective self-assessment of knowledge
and skills, continue learning, refine existing skills and acquire new skills.
3.3.2. Demonstrate ability to apply newly gained knowledge or skills to the care of
the patient.
3.3.3. Demonstrate ability to introspect and utilize experiences, to enhance
personal and professional growth and learning.
3.3.4. Demonstrate ability to search (including through electronic means), and
critically evaluate the medical literature and apply the information in the
care of the patient.
3.3.5. Be able to identify and select an appropriate career pathway that is
professionally rewarding and personally fulfilling.
3.5. Professional who is committed to excellence, is ethical, responsive
and accountable to patients, community and the profession
3.5.1 Practice selflessness, integrity, responsibility, accountability and respect.
3.5.2 Respect and maintain professional boundaries between patients, colleagues
and society.
3.5.3. Demonstrate ability to recognize and manage ethical and professional
conflicts.
3.5.4. Abide by prescribed ethical and legal codes of conduct and practice.
3.5.5. Demonstrate a commitment to the growth of the medical profession as a
whole.
4. Broad Outline on training format
4.1. In order to ensure that training is in alignment with the goals and competencies
listed in sub-clause 2 and 3 above:
4.1.1. There shall be a "Foundation Course" to orient medical learners to MBBS
programme, and provide them with requisite knowledge, communication
(including electronic), technical and language skills.
4.1.2. The curricular contents shall be vertically and horizontally aligned and
integrated to the maximum extent possible in order to enhance learner’s
interest and eliminate redundancy and overlap.
9

4.1.3. Teaching-learning methods shall be learner centric and shall predominantly


include small group learning, interactive teaching methods and case based
learning.

4.1.4. Clinical training shall emphasize early clinical exposure, skill acquisition,
certification in essential skills; community/primary/secondary care-based
learning experiences and emergencies.

4.1.5. Training shall primarily focus on preventive and community based


approaches to health and disease, with specific emphasis on national health
priorities such as family welfare, communicable and non- communicable
diseases including cancer, epidemics and disaster management.
4.1.6.1. Acquisition and certification of skills shall be through experiences in patient
care, diagnostic and skill laboratories.
4.1.7. The development of ethical values and overall professional growth as integral
part of curriculum shall be emphasized through a structured longitudinal and
dedicated programme on professional development including attitude, ethics
and communication.
4.1.8. Progress of the medical learner shall be documented through structured
periodic assessment that includes formative and summative assessments.
Logs of skill-based training shall be also maintained.
4.1.9. Appropriate Faculty Development Programmes shall be conducted regularly
by institutions to facilitate medical teachers at all levels to continuously
update their professional and teaching skills, and align their teaching skills
to curricular objectives.
10

CHAPTER II
ADMISSION TO INDIAN MEDICAL GRADUATE PROGRAMME:
NATIONAL ELIGIBILITY-CUM- ENTRANCE TEST AND
COMMON COUNSELLING
5. Admission to the Indian Medical Graduate Programme
The provision as contained in Part I – Chapter II shall be the governing provisions.

CHAPTER III
MIGRATION
6. Migration

The provision as contained in Part I - Chapter II Clause 6 shall be the governing


provisions.
CHAPTER IV
PHASE WISE TRAINING AND TIME DISTRIBUTION FOR
PROFESSIONAL DEVELOPMENT
The Competency based Undergraduate Curriculum and Attitude, Ethics and Communication
(AETCOM) course, as published by the Medical Council of India and also made available
on the Council’s website, shall be the curriculum for the batches admitted in MBBS from
the academic year 2019-20 onwards.

Provided that in respect of batches admitted prior to the academic year 2019-20, the
governing provisions shall remain as contained in the Part I of these Regulations.
7. Training period and time distribution:
7.1. Every learner shall undergo a period of certified study extending over 4 ½
academic years, divided into nine semesters from the date of commencement
of course to the date of completion of examination which shall be followed by
one year of compulsory rotating internship.
7.2. Each academic year will have at least 240 teaching days with a minimum of
eight hours of working on each day including one hour as lunch break.
7.3. Teaching and learning shall be aligned and integrated across specialties both
vertically and horizontally for better learner comprehension. Learner centered
learning methods should include problem oriented learning, case studies,
community oriented learning, self- directed and experiential learning.
7.4. The period of 4 ½ years is divided as follows:
7.4.1. Pre-Clinical Phase [(Phase I) - First Professional phase of 13 months preceded
by Foundation Course of one month]: will consist of preclinical subjects –
Human Anatomy, Physiology, Biochemistry, Introduction to Community
11

Medicine, Humanities, Professional development including Attitude, Ethics &


Communication (AETCOM) module and early clinical exposure, ensuring both
horizontal and vertical integration.
7.4.2. Para-clinical phase [(Phase II) - Second Professional (12 months)]: will consist
of Para-clinical subjects namely Pathology, Pharmacology, Microbiology,
Community Medicine, Forensic Medicine and Toxicology, Professional
development including Attitude, Ethics & Communication (AETCOM) module
and introduction to clinical subjects ensuring both horizontal and vertical
integration.

The clinical exposure to learners will be in the form of learner-doctor method


of clinical training in all phases. The emphasis will be on primary, preventive
and comprehensive health care. A part of training during clinical postings
should take place at the primary level of health care. It is desirable to provide
learning experiences in secondary health care, wherever possible. This will
involve:
(a) Experience in recognizing and managing common problems seen in
outpatient, inpatient and emergency settings,
(b) Involvement in patient care as a team member,
(c) Involvement in patient management and performance of basic
procedures.
7.4.3. Clinical Phase – [(Phase III) Third Professional (28 months)]
(a) Part I (13 months) - The clinical subjects include General Medicine,
General Surgery, Obstetrics & Gynaecology, Pediatrics, Orthopaedics,
Dermatology, Otorhinolaryngology, Ophthalmology, Community
Medicine, Forensic Medicine and Toxicology, Psychiatry, Respiratory
Medicine, Radiodiagnosis & Radiotherapy and Anaesthesiology &
Professional development including AETCOM module.
(b) Electives (2 months) - To provide learners with opportunity for diverse
learning experiences, to do research/community projects that will
stimulate enquiry, self directed experimental learning and lateral thinking
[9.3].
(c) Part II (13 months) - Clinical subjects include:
i. Medicine and allied specialties (General Medicine, Psychiatry,
Dermatology Venereology and Leprosy (DVL), Respiratory Medicine
including Tuberculosis)
ii. Surgery and allied specialties (General Surgery, Orthopedics
[including trauma]), Dentistry, Physical Medicine and rehabilitation,
Anaesthesiology and Radiodiagnosis)
iii. Obstetrics and Gynecology (including Family Welfare)
iv. Pediatrics
v. AETCOM module
12

7.4. Didactic lectures shall not exceed one third of the schedule; two third of the
schedule shall include interactive sessions, practicals, clinical or/and group
discussions. The learning process should include clinical experiences, problem
oriented approach, case studies and community health care activities.
The admission shall be made strictly in accordance with the statutory notified
time schedule towards the same.
7.5. Universities shall organize admission timing and admission process in such a
way that teaching in the first Professional year commences with induction
through the Foundation Course by the 1st of August of each year.
(i) Supplementary examinations shall not be conducted later than 90 days
from the date of declaration of the results of the main examination, so that
the learners who pass can join the main batch for progression and the
remainder would appear for the examination in the subsequent year.
(ii) A learner shall not be entitled to graduate later than ten (10) years of her/his
joining the first MBBS course.
7.6. No more than four attempts shall be allowed for a candidate to pass the first
Professional examination. The total period for successful completion of first
Professional course shall not exceed four (4) years. Partial attendance of
examination in any subject shall be counted as an attempt.
7.7. A learner, who fails in the second Professional examination, shall not be
allowed to appear in third Professional Part I examination unless she/he
passes all subjects of second Professional examination.
7.8. Passing in third Professional (Part I) examination is not compulsory before
starting part II training; however, passing of third Professional (Part I) is
compulsory for being eligible for third Professional (Part II) examination.
7.9. During para-clinical and clinical phases, including prescribed 2 months of
electives, clinical postings of three hours duration daily as specified in
Tables 5, 6, 7 and 8 would apply for various departments.
7.10. Passing in first Professional is compulsory before proceeding to phase II
training
8. Phase distribution and timing of examination
8.1. Time distribution of the MBBS programme is given in Table 1.
8.2. Distribution of subjects by Professional Phase-wise is given in Table 2.
8.3. Minimum teaching hours prescribed in various disciplines are as under
Tables 3-7.
8.4. Distribution of clinical postings is given in Table 8.
8.5. Duration of clinical postings will be:
8.5.1. Second Professional : 36 weeks of clinical posting (Three hours per day - five
days per week : Total 540 hours)

8.5.2. Third Professional part I: 42 weeks of clinical posting (Three hours per day -
six days per week : Total 756 hours)
13

8.5.3. Third Professional part II: 44 weeks of clinical posting (Three hours per day -
six days per week : Total 792 hours)
8.6. Time allotted excludes time reserved for internal / University examinations,
and vacation.
8.7. Second professional clinical postings shall commence before / after declaration
of results of the first professional phase examinations, as decided by the
institution/ University. Third Professional parts I and part II clinical postings
shall start no later than two weeks after the completion of the previous
professional examination.
8.8. 25% of allotted time of third Professional shall be utilized for integrated
learning with pre- and para- clinical subjects. This will be included in the
assessment of clinical subjects.

Table 1: Time distribution of MBBS Programme & Examination Schedule

Jan Feb Mar Apr May Jun Jul Aug Sep Oct Nov Dec

Foundation I MBBS
Course
I MBBS Exam I II MBBS
MBBS
II MBBS Exam II III MBBS
MBBS
Exam III
III MBBS MBBS Electives &
Part I
Part I Skills
III MBBS Part II

Exam
III Internship
MBBS
Part II

Internship

• One month is provided at the end of every professional year for completion of
examination and declaration of results.
14

Table 2: Distribution of subjects by Professional Phase


Phase & year of Duration# University
MBBS training Subjects & New Teaching examination
Elements
Foundation Course (1 month)
First Professional Human Anatomy, Physiology &
MBBS Biochemistry, introduction to 1 + 13 months I Professional
Community Medicine, Humanities
Early Clinical Exposure
Attitude, Ethics, and Communication
Module (AETCOM)
• Pathology, Microbiology,
Pharmacology, Forensic
Second Professional Medicine and Toxicology,
12 months II Professional
MBBS • Introduction to clinical subjects
including Community Medicine
• Clinical postings
• Attitude, Ethics & Communication
Module (AETCOM)
• General Medicine, General
Surgery, Obstetrics &
Third Professional Gynecology, Pediatrics,
13 months III
MBBS Part I Orthopedics, Dermatology,
Psychiatry, Professiona
Otorhinolaryngology, l (Part I)
Ophthalmology, Community
Medicine, Forensic Medicine
and Toxicology, Respiratory
medicine, Radiodiagnosis &
Radiotherapy, Anesthesiology
• Clinical subjects /postings
• Attitude, Ethics & Communication
Module (AETCOM)
Electives • Electives, Skills and assessment* 2 months
• General Medicine, Pediatrics,
General Surgery, Orthopedics,
Third Professional 13 months III
Obstetrics and Gynecology
MBBS Part II including Family welfare and Professional
allied specialties
(Part II)
• Clinical postings/subjects

Attitude, Ethics & Communication
Module (AETCOM)
*Assessment of electives shall be included in Internal Assessment.
15

Table 3: Foundation Course (one month)

Teachin g Self Directed


Subjects/ Contents hours Total
Learning hours
(hours)
Orientation 1 3 0 30
0
Skills Module 2 3 0 35
5
Field visit to Community Health Center 8 0 8
Introduction to Professional Development & - - 40
AETCOM module
Sports and extracurricular activities 2 0 22
2
Enhancement of language/ computer skills 3 4 0 40
0
- - 175

1. Orientation course will be completed as single block in the first week and will
contain elements outlined in 9.1.
2. Skills modules will contain elements outlined in 9.1.
3. Based on perceived need of learners, one may choose language enhancement
(English or local spoken or both) and computer skills. This should be provided
longitudinally through the duration of the Foundation Course.
Teaching of Foundation Course will be organized by pre-clinical departments.

Table 4: First Professional teaching hours


Lectures Small Group Self Total
Subjects (hours) Teaching/ Tutorials/ directed
Integrated learning/ learning (hours)
Practical (hours) (hours)

Human Anatomy 220 41 40 675


5
Physiology* 160 31 25 495
0
Biochemistry 80 15 20 250
0
Early Clinical Exposure** 90 - 0 90
Community Medicine 20 27 5 52
Attitude, Ethics & - 26 8 34
Communication Module
(AETCOM) ***
Sports and extracurricular - - - 60
activities
16

Formative assessment and Term - - - 80


examinations
Total - - - 1736

* including Molecular Biology.


** Early clinical exposure hours to be divided equally in all three subjects.
*** AETCOM module shall be a longitudinal programme.
Table 5: Second Professional teaching hours
Small group Clinical Self -
Subjects Lectures learning Directed Total
(Tutorials / Postings Learning (hours)
(hours) Seminars) (hours) * (hours)
/Integrated
learning
(hours)

Pathology 80 138 - 12 230


Pharmacology 80 138 - 12 230
Microbiology 70 110 - 10 190

Community Medicine 20 30 - 10 60
Forensic Medicine and 15 30 - 5 50
Toxicology
Clinical Subjects 75** - 540*** 615
Attitude, Ethics & 29 - 8 37
Communication Module
(AETCOM)
Sports and - - - 28 28
extracurricular activities
Total - - - - 1440

* At least 3 hours of clinical instruction each week must be allotted to training in


clinical and procedural skill laboratories. Hours may be distributed weekly or as a
block in each posting based on institutional logistics.
** 25 hours each for Medicine, Surgery and Gynecology & Obstetrics.
***The clinical postings in the second professional shall be 15 hours per week (3 hrs
per day from Monday to Friday).
17

Table 6: Third Professional Part I teaching hours

Teaching Tutorials/ Self- Total


Subjects Seminars Directed
Hours Learning (hours)
/Integrated
Teaching (hours)
(hours)
General Medicine 25 35 5 65
General Surgery 25 35 5 65
Obstetrics and Gynecology 25 35 5 65
Pediatrics 20 30 5 55
Orthopaedics 15 20 5 40
Forensic Medicine and Toxicology 25 45 5 75
Community Medicine 40 60 5 105
Dermatology 20 5 5 30
Psychiatry 25 10 5 40
Respiratory Medicine 10 8 2 20
Otorhinolaryngology 25 40 5 70
Ophthalmology 30 60 10 100
Radiodiagnosis and Radiotherapy 10 8 2 20
Anesthesiology 8 10 2 20
Clinical Postings* - - - 756
Attitude, Ethics & 19 06 25
Communication Module
(AETCOM)
Total 303 401 66 155
1
* The clinical postings in the third professional part I shall be 18 hours per week (3 hrs per day
from Monday to Saturday).
Table 7: Third Professional Part II teaching hours

Teaching Tutorials/Sem Self - Total*


Subjects inars / Integrated Directed
Hours Teaching Learning (hours)
(hours) (hours)

General Medicine 70 125 15 210


General Surgery 70 125 15 210
Obstetrics and Gynecology 70 125 15 210
Pediatrics 20 35 10 65
Orthopaedics 20 25 5 50
18

Clinical Postings** 792


Attitude, Ethics & 28 16 43
Communication Module
(AETCOM)***
Electives 200
Total 250 435 60 1780

* 25% of allotted time of third professional shall be utilized for integrated learning with
pre- and para- clinical subjects and shall be assessed during the clinical subjects
examination. This allotted time will be utilized as integrated teaching by para- clinical
subjects with clinical subjects (as Clinical Pathology, Clinical Pharmacology and
Clinical Microbiology).
** The clinical postings in the third professional part II shall be 18 hours per week (3 hrs per day
from Monday to Saturday).

*** Hours from clinical postings can also be used for AETCOM modules.
Table 8: Clinical postings
Period of training in weeks
Total
Subjects weeks
II III MBBS III MBBS
MBBS Part I
Part II

Electives - - 8* (4 regular 4
clinical
posting)
General Medicine1 4 4 8+4 20
General Surgery 4 4 8+4 20
Obstetrics &Gynaecology2 4 4 8 +4 20
Pediatrics 2 4 4 10
Community Medicine 4 6 - 10
Orthopedics - including Trauma3 2 4 2 8
Otorhinolaryngology 4 4 - 8
Ophthalmology 4 4 - 8
Respiratory Medicine 2 - - 2
Psychiatry 2 2 - 4
Radiodiagnosis4 2 - - 2
Dermatology, Venereology & 2 2 2 6
Leprosy
Dentistry & Anesthesia - 2 - 2
Casualty - 2 - 2
36 4 48 126
2
19

* In four of the eight weeks of electives, regular clinical postings shall be accommodated.
Clinical postings may be adjusted within the time framework.
1 This posting includes Laboratory Medicine (Para-clinical) & Infectious Diseases
(Phase III Part I).
2 This includes maternity training and family welfare (including Family Planning).

3 This posting includes Physical Medicine and Rehabilitation.

4 This posting includes Radiotherapy, wherever available.

9. New teaching / learning elements


9.1. Foundation Course
9.2. Goal: The goal of the Foundation Course is to prepare a learner to study
medicine effectively. It will be of one month duration after admission.
9.2.1. Objectives: The objectives are to:
(a) Orient the learner to:
(i) The medical profession and the physician’s role in society
(ii) The MBBS programme
(iii) Alternate health systems in the country and history of medicine
(iv) Medical ethics, attitudes and professionalism
(v) Health care system and its delivery
(vi) National health programmes and policies
(vii) Universal precautions and vaccinations
(viii) Patient safety and biohazard safety
(ix) Principles of primary care (general and community based care)
(x) The academic ambience
(b) Enable the learner to acquire enhanced skills in:
(i) Language
(ii) Interpersonal relationships
(iii) Communication
(iv) Learning including self-directed learning
(v) Time management
(vi) Stress management
(vii) Use of information technology
(c) Train the learner to provide:
a. First-aid
b. Basic life support
20

9.1.3. In addition to the above, learners may be enrolled in one of the following
programmes which will be run concurrently:
(a) Local language programme
(b) English language programme
(c) Computer skills
(d) These may be done in the last two hours of the day for the duration of the
Foundation Course.
9.1.4. These sessions must be as interactive as possible.
9.1.5. Sports (to be used through the Foundation Course as protected 04 hours /
week).
9.1.6. Leisure and extracurricular activity (to be used through the Foundation
Course as protected 02 hours per week).
9.1.7. Institutions shall develop learning modules and identify the appropriate
resource persons for their delivery.
9.1.8. The time committed for the Foundation Course may not be used for any other
curricular activity.
9.1.9. The Foundation Course will have compulsory 75% attendance. This will be
certified by the Dean of the college.
9.1.10. The Foundation Course will be organized by the Coordinator appointed by the
Dean of the college and will be under supervision of the heads of the preclinical
departments.
9.1.11. Every college must arrange for a meeting with parents and their wards.
9.2. Early Clinical Exposure
9.2.1. Objectives: The objectives of early clinical exposure of the first-year medical
learners are to enable the learner to:
(a) Recognize the relevance of basic sciences in diagnosis, patient care and
treatment,
(b) Provide a context that will enhance basic science learning,
(c) Relate to experience of patients as a motivation to learn,
(d) Recognize attitude, ethics and professionalism as integral to the doctor-
patient relationship,
(e) Understand the socio-cultural context of disease through the study of
humanities.
9.2.2. Elements
(a) Basic science correlation: i.e. apply and correlate principles of basic sciences
as they relate to the care of the patient (this will be part of integrated modules).
(b) Clinical skills: to include basic skills in interviewing patients, doctor-patient
communication, ethics and professionalism, critical thinking and analysis and
self-learning (this training will be imparted in the time allotted for early
clinical exposure).
21

(c) Humanities: To introduce learners to a broader understanding of the socio-


economic framework and cultural context within which health is delivered
through the study of humanities and social sciences.
9.3. Electives
9.3.1. Objectives: To provide the learner with opportunities:
(a) For diverse learning experiences,
(b) To do research/community projects that will stimulate enquiry, self-
directed, experiential learning and lateral thinking.
9.3.2. Two months are designated for elective rotations after completion of the
examination at end of the third MBBS Part I and before commencement of
third MBBS Part II.
9.3.3. It is mandatory for learners to do an elective. The elective time should not be
used to make up for missed clinical postings, shortage of attendance or other
purposes.
9.3.4. Structure
(a) The learner shall rotate through two elective blocks of 04 weeks each.
(b) Block 1 shall be done in a pre-selected preclinical or para-clinical or other
basic sciences laboratory OR under a researcher in an ongoing research
project.
During the electives regular clinical postings shall continue.
(c) Block 2 shall be done in a clinical department (including specialties, super-
specialties, ICUs, blood bank and casualty) from a list of electives
developed and available in the institution.
OR
as a supervised learning experience at a rural or urban community clinic.
(d) Institutions will pre-determine the number and nature of electives,
names of the supervisors, and the number of learners in each elective
based on the local conditions, available resources and faculty.
9.3.5. Each institution will develop its own mechanism for allocation of electives.
9.3.6. It is preferable that elective choices are made available to the learners in the
beginning of the academic year.
9.3.7. The learner must submit a learning log book based on both blocks of the
elective.
9.3.8. 75% attendance in the electives and submission of log book maintained during
elective is required for eligibility to appear in the final MBBS examination.
9.3.9. Institutions may use part of this time for strengthening basic skill
certification.
22

9.4. Professional Development including Attitude, Ethics and Communication Module


(AETCOM)
9.4.1. Objectives of the programme: At the end of the programme, the learner must
demonstrate ability to:
(a) understand and apply principles of bioethics and law as they apply to
medical practice and research understand and apply the principles of
clinical reasoning as they apply to the care of the patients,
(b) understand and apply the principles of system based care as they relate to
the care of the patient,
(c) understand and apply empathy and other human values to the care of the
patient,
(d) communicate effectively with patients, families, colleagues and other
health care professionals,
(e) understand the strengths and limitations of alternative systems of
medicine,
(f) respond to events and issues in a professional, considerate and humane
fashion,
(g) translate learning from the humanities in order to further his / her
professional and personal growth.
9.4.2. Learning experiences:
(a) This will be a longitudinal programme spread across the continuum
of the MBBS programme including internship,
(b) Learning experiences may include – small group discussions, patient
care scenarios, workshop, seminars, role plays, lectures etc.
(c) Attitude, Ethics &Communication Module (AETCOM module)
developed by Medical Council of India should be used longitudinally
for purposes of instruction.
9.4.3. 75% attendance in Professional Development Programme (AETCOM
Module) is required for eligibility to appear for final examination in each
professional year.
9.4.4. Internal Assessment will include:
(a) Written tests comprising of short notes and creative writing experiences,
(b) OSCE based clinical scenarios / viva voce.
9.4.5. At least one question in each paper of the clinical specialties in the University
examination should test knowledge competencies acquired during the
professional development programme.
9.4.6. Skill competencies acquired during the Professional Development
Programme must be tested during the clinical, practical and viva voce.
9.5. Learner-doctor method of clinical training (Clinical Clerkship)
23

9.5.1. Goal: To provide learners with experience in:


(a) Longitudinal patient care,
(b) Being part of the health care team,
(c) Hands-on care of patients in outpatient and inpatient setting.
9.5.2. Structure:
(a) The first clinical posting in second professional shall orient learners to
the patient, their roles and the specialty.
(b) The learner-doctor programme will progress as outlined in Table 9.
(c) The learner will function as a part of the health care team with the
following responsibilities:
(i) Be part of the unit’s outpatient services on admission days,
(ii) Remain with the admission unit until 6 PM except during designated
class hours,
(iii) Be assigned patients admitted during each admission day for whom
he/she will undertake responsibility, under the supervision of a
senior resident or faculty member,
(iv) Participate in the unit rounds on its admission day and will
present the assigned patients to the supervising physician,
(v) Follow the patient’s progress throughout the hospital stay until
discharge,
(vi) Participate, under supervision, in procedures, surgeries, deliveries
etc. of assigned patients (according to responsibilities outlined in
table 9),
(vii) Participate in unit rounds on at least one other day of the week
excluding the admission day,
(viii) Discuss ethical and other humanitarian issues during unit rounds,
(ix) Attend all scheduled classes and educational activities,
(x) Document his/her observations in a prescribed log book / case record.
(d) No learner will be given independent charge of the patient
(e) The supervising physician will be responsible for all patient care decisions
9.5.3. Assessment:
(a) A designated faculty member in each unit will coordinate and facilitate the
activities of the learner, monitor progress, provide feedback and review the
log book/ case record.
(b) The log book/ case record must include the written case record prepared
by the learner including relevant investigations, treatment and its
rationale, hospital course, family and patient discussions, discharge
summary etc.
24

(c) The log book should also include records of outpatients assigned.
Submission of the log book/ case record to the department is required for
eligibility to appear for the final examination of the subject.
Table 9: Learner - Doctor programme (Clinical Clerkship)

Year of
Curriculum Focus of Learner - Doctor programme
Year 1 Introduction to hospital environment, early clinical exposure,
understanding perspectives of illness
Year 2 History taking, physical examination, assessment of change in clinical
status, communication and patient education
Year 3 All of the above and choice of investigations, basic procedures and
continuity of care
Year 4 All of the above and decision making, management and outcomes
25

CHAPTER V
COMPETENCY BASED CURRICULUM OF THE INDIAN MEDICAL GRADUATE
PROGRAMME
10. Specific Competencies
10.1. Preamble: The salient feature of the revision of the medical curriculum in
2019 is the emphasis on learning which is competency-based, integrated and
learner-centered acquisition of skills and ethical & humanistic values.
Each of the competencies described below must be read in conjunction with the
goals of the medical education as listed in items 2 to 3.5.5
It is recommended that didactic teaching be restricted to less than one third of the
total time allotted for that discipline. Greater emphasis is to be laid on hands- on
training, symposia, seminars, small group discussions, problem-oriented and
problem-based discussions and self-directed learning. Learners must be
encouraged to take active part in and shared responsibility for their learning.
The global competencies to be achieved by the learner are outlined above in
Chapter 1- section 3. Since the MBBS programme assessment will continue to be
subject based, subject specific competencies have been outlined. These have to be
acquired by the learner in the corresponding professional year. These
competencies must be interpreted in the larger context outlined in section 3 and
may be considered as “sub competencies” of the global competencies.
10.2. Integration must be horizontal (i.e. across disciplines in a given phase of the
course) and vertical (across different phases of the course). As far as possible, it
is desirable that teaching/learning occurs in each phase through study of organ
systems or disease blocks in order to align the learning process. Clinical cases
must be used to integrate and link learning across disciplines.
10.3. Pre-clinical Subjects
10.3.1. Human Anatomy
a) Competencies: The undergraduate must demonstrate:

1. Understanding of the gross and microscopic structure and development


of human body,
2. Comprehension of the normal regulation and integration of the functions
of the organs and systems on basis of the structure and genetic pattern,
3. Understanding of the clinical correlation of the organs and structures
involved and interpret the anatomical basis of the disease presentations.
(b) Integration: The teaching should be aligned and integrated horizontally and
vertically in organ systems with clinical correlation that will provide a context
for the learner to understand the relationship between structure and function
and interpret the anatomical basis of various clinical conditions and
procedures.
26

10.3.2. Physiology
(a) Competencies: The undergraduates must demonstrate:
1. Understanding of the normal functioning of the organs and organ
systems of the body,
2. Comprehension of the normal structure and organization of the organs
and systems on basis of the functions,
3. Understanding of age-related physiological changes in the organ
functions that reflect normal growth and development,
4. Understand the physiological basis of diseases.
(b) Integration: The teaching should be aligned and integrated horizontally and
vertically in organ systems in order to provide a context in which normal
function can be correlated both with structure and with the biological basis,
its clinical features, diagnosis and therapy.
10.3.3. Biochemistry
The course will comprise Molecular and Cellular Biochemistry.

(a)Competencies: The learner must demonstrate an understanding of:


1. Biochemical and molecular processes involved in health and disease,
2. Importance of nutrition in health and disease,
3. Biochemical basis and rationale of clinical laboratory tests,and
demonstrate ability to interpret these in the clinical context.
(b) Integration: The teaching/learning programme should be integrated
horizontally and vertically, as much as possible, to enable learners to make
clinical correlations and to acquire an understanding of the cellular and
molecular basis of health and disease.
10.3.4. Introduction to Community Medicine
(a) Competencies: The undergraduate must demonstrate:
1. Understanding of the concept of health and disease,
2. Understanding of demography, population dynamics and disease burden
in National and global context,
3. Comprehension of principles of health economics and hospital
management,
4. Understanding of interventions to promote health and prevent diseases
as envisioned in National and State Health Programmes.
10.4. Second Professional (Para-Clinical)
10.4.1. Pathology
(a) Competencies: The undergraduate must demonstrate:
1. Comprehension of the causes, evolution and mechanisms of diseases,
2. Knowledge of alterations in gross and cellular morphology of organs in
disease states,
3. Ability to correlate the natural history, structural and functional
changes with the clinical manifestations of diseases, their diagnosis
and therapy,
27

(b) Integration: The teaching should be aligned and integrated


horizontally and vertically in organ systems recognizing deviations from
normal structure and function and clinically correlated so as to provide
an overall understanding of the etiology, mechanisms, laboratory
diagnosis, and management of diseases.
10.4.2. Microbiology
(a) Competencies: The undergraduate learner demonstrate:
1. Understanding of role of microbial agents in health and disease,
2. Understanding of the immunological mechanisms in health and
disease,
3. Ability to correlate the natural history, mechanisms and clinical
manifestations of infectious diseases as they relate to the properties
of microbial agents,
4. Knowledge of the principles and application of infection control
measures,
5. An understanding of the basis of choice of laboratory diagnostic tests
and their interpretation, antimicrobial therapy, control and
prevention of infectious diseases.
(b) Integration: The teaching should be aligned and integrated
horizontally and vertically in organ systems with emphasis on host-
microbe-environment interactions and their alterations in disease and
clinical correlations so as to provide an overall understanding of the
etiological agents, their laboratory diagnosis and prevention.
10.4.3. Pharmacology
(a) Competencies: The undergraduate must demonstrate:
1. Knowledge about essential and commonly used drugs and an
understanding of the pharmacologic basis of therapeutics,
2. Ability to select and prescribe medicines based on clinical condition
and the pharmacologic properties, efficacy, safety, suitability and cost
of medicines for common clinical conditions of national importance,
3. Knowledge of pharmacovigilance, essential medicine concept and
sources of drug information and industry-doctor relationship,
4. Ability to counsel patients regarding appropriate use of prescribed
drug and drug delivery systems.
(b) Integration: The teaching should be aligned and integrated
horizontally and vertically in organ systems recognizing the interaction
between drug, host and disease in order to provide an overall
understanding of the context of therapy.
28

10.4.4. Forensic Medicine and Toxicology


(a) Competencies: The learner must demonstrate:
1. Understanding of medico-legal responsibilities of physicians in
primary and secondary care settings,
2. Understanding of the rational approach to the investigation of crime,
based on scientific and legal principles,
3. Ability to manage medical and legal issues in cases of poisoning /
overdose,
4. Understanding the medico-legal framework of medical practice and
medical negligence,
5. Understanding of codes of conduct and medical ethics.
(b) Integration: The teaching should be aligned and integrated horizontally
and vertically recognizing the importance of medico-legal, ethical and
toxicological issues as they relate to the practice of medicine.
10.4.5. Community Medicine – as per 10.3.4
10.5. Third Professional (Part I)
10.5.1. General Medicine
(a) Competencies: The student must demonstrate ability to do the following
in relation to common medical problems of the adult in the community:
1. Demonstrate understanding of the patho-physiologic basis,
epidemiological profile, signs and symptoms of disease and their
investigation and management,
2. Competently interview and examine an adult patient and make a
clinical diagnosis,
3. Appropriately order and interpret laboratory tests,
4. Initiate appropriate cost-effective treatment based on an
understanding of the rational drug prescriptions, medical
interventions required and preventive measures,
5. Follow up of patients with medical problems and refer whenever
required,
6. Communicate effectively, educate and counsel the patient and
family,
7. Manage common medical emergencies and refer when required,
8. Independently perform common medical procedures safely and
understand patient safety issues.
(b) Integration: The teaching should be aligned and integrated horizontally
and vertically in order to provide sound biologic basis and incorporating
the principles of general medicine into a holistic and comprehensive
approach to the care of the patient.
29

10.5.2. General Surgery


(a) Competencies: The student must demonstrate:
1. Understanding of the structural and functional basis, principles of
diagnosis and management of common surgical problems in adults
and children,
2. Ability to choose, calculate and administer appropriately intravenous
fluids, electrolytes, blood and blood products based on the clinical
condition,
3. Ability to apply the principles of asepsis, sterilization, disinfection,
rational use of prophylaxis, therapeutic utilities of antibiotics and
universal precautions in surgical practice,
4. Knowledge of common malignancies in India and their prevention,
early detection and therapy,
5. Ability to perform common diagnostic and surgical procedures at the
primary care level,
6. Ability to recognize, resuscitate, stabilize and provide Basic &
Advanced Life Support to patients following trauma,
7. Ability to administer informed consent and counsel patient prior to
surgical procedures,
8. Commitment to advancement of quality and patient safety in surgical
practice.
(b) Integration: The teaching should be aligned and integrated
horizontally and vertically in order to provide a sound biologic basis
and a holistic approach to the care of the surgical patient.
10.5.3. Obstetrics and Gynaecology
(a) Competencies in Obstetrics: The student must demonstrate ability to:
1. Provide peri-conceptional counseling and antenatal care,
2. Identify high-risk pregnancies and refer appropriately,
3. Conduct normal deliveries, using safe delivery practices in the
primary and secondary care settings,
4. Prescribe drugs safely and appropriately in pregnancy and lactation,
5. Diagnose complications of labor, institute primary care and refer in a
timely manner,
6. Perform early neonatal resuscitation,
7. Provide postnatal care, including education in breast-feeding,
8. Counsel and support couples in the correct choice of contraception,
9. Interpret test results of laboratory and radiological investigations as
they apply to the care of the obstetric patient,
10. Apply medico-legal principles as they apply to tubectomy, Medical
30

Termination of Pregnancy (MTP), Pre-conception and Prenatal


Diagnostic Techniques (PC PNDT Act) and other related Acts.
Competencies in Gynecology: The student must demonstrate ability to:
1. Elicit a gynecologic history, perform appropriate physical and pelvic
examinations and PAP smear in the primary care setting,
2. Recognize, diagnose and manage common reproductive tract
infections in the primary care setting,
3. Recognize and diagnose common genital cancers and refer them
appropriately.
(b) Integration: The teaching should be aligned and integrated
horizontally and vertically in order to provide comprehensive care for
women in their reproductive years and beyond, based on a sound
knowledge of structure, functions and disease and their clinical, social,
emotional, psychological correlates in the context of national health
priorities.
10.5.4. Pediatrics
(a) Competencies: The student must demonstrate:
1. Ability to assess and promote optimal growth, development and
nutrition of children and adolescents and identify deviations from
normal,
2. Ability to recognize and provide emergency and routine ambulatory
and First Level Referral Unit care for neonates, infants, children and
adolescents and refer as may be appropriate,
3. Ability to perform procedures as indicated for children of all ages in
the primary care setting,
4. Ability to recognize children with special needs and refer
appropriately,
5. Ability to promote health and prevent diseases in children,
6. Ability to participate in National Programmes related to child health
and in conformation with the Integrated Management of Neonatal
and Childhood Illnesses (IMNCI) Strategy,
7. Ability to communicate appropriately and effectively.
(b) Integration: The teaching should be aligned and integrated
horizontally and vertically in order to provide comprehensive care for
neonates, infants, children and adolescents based on a sound
knowledge of growth, development, disease and their clinical, social,
emotional, psychological correlates in the context of national health
priorities.
31

10.5.5. Orthopaedics (including Trauma)


(a) Competencies: The student must demonstrate:
1. Ability to recognize and assess bone injuries, dislocation and poly-
trauma and provide first contact care prior to appropriate referral,
2. Knowledge of the medico-legal aspects of trauma,
3. Ability to recognize and manage common infections of bone and
joints in the primary care setting,
4. Recognize common congenital, metabolic, neoplastic, degenerative
and inflammatory bone diseases and refer appropriately,
5. Ability to perform simple orthopaedic techniques as applicable to a
primary care setting,
6. Ability to recommend rehabilitative services for common orthopaedic
problems across all ages.
(b) Integration: The teaching should be aligned and integrated horizontally
and vertically in order to allow the student to understand the structural
basis of orthopaedic problems, their management and correlation with
function, rehabilitation and quality of life.
10.5.6. Forensic Medicine and Toxicology – as per 10.4.4
10.5.7. Community medicine
(a) Competencies: The learner must demonstrate:
1. Understanding of physical, social, psychological, economic and
environmental determinants of health and disease,
2. Ability to recognize and manage common health problems including
physical, emotional and social aspects at individual family and
community level in the context of National Health Programmes,
3. Ability to Implement and monitor National Health Programmes in the
primary care setting,
4. Knowledge of maternal and child wellness as they apply to national
health care priorities and programmes,
5. Ability to recognize, investigate, report, plan and manage
community health problems including malnutrition and
emergencies.
(b) Integration: The teaching should be aligned and integrated
horizontally and vertically in order to allow the learner to understand
the impact of environment, society and national health
priorities as they relate to the promotion of health and prevention and
cure of disease.
10.5.8. Dermatology, Venereology & Leprosy
(a) Competencies: The undergraduate student must demonstrate:
1. Understanding of the principles of diagnosis of diseases of the skin,
hair, nail and mucosa,
2. Ability to recognize, diagnose, order appropriate investigations and
treat common diseases of the skin including leprosy in the primary
care setting and refer as appropriate,
3. A syndromic approach to the recognition, diagnosis, prevention,
32

counseling, testing and management of common sexually


transmitted diseases including HIV based on national health
priorities,
4. Ability to recognize and treat emergencies including drug reactions
and refer as appropriate.
(b) Integration: The teaching should be aligned and integrated horizontally
and vertically in order to emphasize the biologic basis of diseases of
the skin, sexually transmitted diseases and leprosy and to provide an
understanding that skin diseases may be a manifestation of systemic
disease.
10.5.9. Psychiatry
(a) Competencies: The student must demonstrate:
1. Ability to promote mental health and mental hygiene,
2. Knowledge of etiology (bio-psycho-social-environmental
interactions), clinical features, diagnosis and management of
common psychiatric disorders across all ages,
3. Ability to recognize and manage common psychological and
psychiatric disorders in a primary care setting, institute preliminary
treatment in disorders difficult to manage, and refer appropriately,
4. Ability to recognize alcohol/ substance abuse disorders and refer them
to appropriate centers,
5. Ability to assess risk for suicide and refer appropriately,
6. Ability to recognize temperamental difficulties and personality
disorders,
7. Assess mental disability and rehabilitate appropriately,
8. Understanding of National and State programmes that address
mental health and welfare of patients and community.
(b) Integration: The teaching should be aligned and integrated
horizontally and vertically in order to allow the student to understand
bio-psycho-social-environmental interactions that lead to diseases/
disorders for preventive, promotive, curative, rehabilitative services
and medico-legal implications in the care of patients both in family and
community.
10.5.10. Respiratory Medicine
(a) Competencies: The student must demonstrate:
1. Knowledge of common chest diseases, their clinical manifestations,
diagnosis and management,
2. Ability to recognize, diagnose and manage pulmonary tuberculosis
as contemplated in National Tuberculosis Control programme,
3. Ability to manage common respiratory emergencies in primary care
setting and refer appropriately.
33

(b)Integration: The teaching should be aligned and integrated horizontally


and vertically in order to allow the student to recognize diagnose and
treat TB in the context of the society, national health priorities, drug
resistance and co-morbid conditions like HIV.
10.5.11. Otorhinolaryngology
(a) Competencies: The learner must demonstrate:
1. Knowledge of the common Otorhinolaryngological (ENT)
emergencies and problems,
2. Ability to recognize, diagnose and manage common ENT
emergencies and problems in primary care setting,
3. Ability to perform simple ENT procedures as applicable in a primary
care setting,
4. Ability to recognize hearing impairment and refer to the
appropriate hearing impairment rehabilitation programme.
(b) Integration: The teaching should be aligned and integrated horizontally
and vertically in order to allow the learner to understand the structural
basis of ENT problems, their management and correlation with
function, rehabilitation and quality of life.
10.5.12. Ophthalmology
(a) Competencies: The student must demonstrate:
1. Knowledge of common eye problems in the community
2. Recognize, diagnose and manage common eye problems and identify
indications for referral,
3. Ability to recognize visual impairment and blindness in the
community and implement National programmes as applicable in
the primary care setting.
(b) Integration: The teaching should be aligned and integrated horizontally
and vertically in order to allow the student to understand the structural
basis of ophthalmologic problems, their management and correlation
with function, rehabilitation and quality of life.
10.5.13. a. Radiodiagnosis
(a) Competencies: The student must demonstrate:
1. Understanding of indications for various radiological investigations
in common clinical practice,
2. Awareness of the ill effects of radiation and various radiation
protective measures to be employed,
3. Ability to identify abnormalities in common radiological
investigations.
(b) Integration: Horizontal and vertical integration to understand the
fundamental principles of radiologic imaging, anatomic correlation and
their application in diagnosis and therapy.
34

10.5.13.b. Radiotherapy
(a) Competencies: The student must demonstrate understanding of:
1. Clinical presentations of various cancers,
2. Appropriate treatment modalities for various types of malignancies,
3. Principles of radiotherapy and techniques.
(b) Integration: Horizontal and vertical integration to enable basic
understanding of fundamental principles of radio-therapeutic
procedures.
10.5.14. Anaesthesiology
(a) Competencies in Anaesthesiology: The student must demonstrate ability
to:
1. Describe and discuss the pre-operative evaluation, assessing fitness
for surgery and the modifications in medications in relation to
anaesthesia / surgery,
2. Describe and discuss the roles of Anaesthesiologist as a peri-operative
physician including pre-medication, endotracheal intubation, general
anaesthesia and recovery (including variations in recovery from
anaesthesia and anaesthetic complications),
3. Describe and discuss the management of acute and chronic pain,
including labour analgesia,
4. Demonstrate awareness about the maintenance of airway in children
and adults in various situations,
5. Demonstrate the awareness about the indications, selection of cases
and execution of cardio- pulmonary resuscitation in emergencies and
in the intensive care and high dependency units,
6. Choose cases for local / regional anaesthesia and demonstrate the
ability to administer the same,
7. Discuss the implications and obtain informed consent for various
procedures and to maintain the documents.
(b) Integration: The teaching should be aligned and integrated
horizontally and vertically in order to provide comprehensive care for
patients undergoing various surgeries, in patients with pain, in intensive
care and in cardio respiratory emergencies. Integration with the
preclinical department of Anatomy, para- clinical department of
Pharmacology and horizontal integration with any/all surgical
specialities is proposed.
10.6. Third Professional (Part II)
10.6.1. General Medicine – as per 10.5.1
10.6.2. General Surgery – as per 10.5.2
10.6.3. Obstetrics & Gynaecology – as per 10.5.3
10.6.4. Pediatrics – as per 10.5.4
10.6.5 Orthopaedics – as per 10.5.5
35

CHAPTER 6
ASSESSMENT
11. Assessment

11.1. Eligibility to appear for Professional examinations


I. ATTENDANCE
a. Attendance requirements are 75% in theory and 80% in practical/ clinical for
eligibility to appear for the University examinations in that subject. In subjects
that are taught in more than one phase – the student must have 75% attendance
in theory and 80% in practical in each phase of instruction in that subject.
b. If an examination comprises more than one subject (for e.g., General Surgery and
allied branches), the student must have 75% attendance in each subject and 80%
attendance in each clinical posting.
c. Students who do not have at least 75% attendance in the electives will not be
eligible for the Third Professional - Part II examination.

II. INTERNAL ASSESSMENT


1. Regular periodic examinations shall be conducted throughout the course. There
shall be no less than three internal assessment examinations in each Preclinical
/ Para-clinical subject and no less than two examinations in each clinical subject
in a professional year. An end of posting clinical assessment shall be conducted
for each clinical posting in each professional year.
2. When subjects are taught in more than one phase, the internal assessment must
be done in each phase and must contribute proportionately to final assessment.
For example, General Medicine must be assessed in second Professional, third
Professional Part I and third Professional Part II, independently.
3. Day to day records and log book (including required skill certifications) should be
given importance in internal assessment. Internal assessment should be based on
competencies and skills.
4. The final internal assessment in a broad clinical specialty (e.g., Surgery and allied
specialties etc.) shall comprise of marks from all the constituent specialties. The
proportion of the marks for each constituent specialty shall be determined by the
time of instruction allotted to each.
5. Learners must secure at least 50% marks of the total marks (combined in theory
and practical / clinical; not less than 40 % marks in theory and practical
separately) assigned for internal assessment in a particular subject in order to be
eligible for appearing at the final University examination of that subject. Internal
assessment marks will reflect as separate head of passing at the summative
examination.
6. The results of internal assessment should be displayed on the notice board within
15 days of the test. Universities shall guide the colleges regarding formulating
36

policies for remedial measures for students who are either not able to score
qualifying marks or have missed on some assessments due to any reason.
7. Learners must have completed the required certifiable competencies for that
phase of training and completed the log book appropriate for that phase of training
to be eligible for appearing at the final university examination of that subject.
8. Students are expected to have required competencies for that phase of training as a
condition for appearing in the Final University Examinations. Possession of required
competencies by the student should be certified by the HOD of that subject and Dean
of the College and supported by the completed Log Book. They are all subject to the
scrutiny by the University.
9. Average of all internal assessment examinations marks, including model exam, is to
be taken for calculating final internal assessment marks.
10. The minimum number of internal assessment examinations to be held in each
professional phase is mentioned in Table 1 and Annexure 1
Table 1: Minimum number of Internal Assessment examinations

PHASE SUBJECTS Exams

I MBBS Anatomy 3
Physiology 3
Biochemistry 3
Community Medicine 1
II MBBS Pathology 3
Pharmacology 3
Microbiology 3
General Medicine 2
General Surgery 2
Obstetrics & Gynaecology 2
Forensic Medicine & Toxicology 2
Community Medicine 2
III (Part 1) MBBS Forensic Medicine & Toxicology 2
Community Medicine 2
Ophthalmology 2
Otorhinolaryngology 2
General Medicine 2
General Surgery 2
Obstetrics & Gynaecology 2
Pediatrics 2
III (Part 2) MBBS General Medicine 2
General Surgery 2
Obstetrics & Gynaecology 2
Pediatrics 2
37

11. Final internal assessment calculation is as follows:


The Theory examination constitutes 50 marks & the Practical/ Clinical 50 marks.

Viva, if included, is to be for a maximum of 10 marks and is added to practical/


clinical. Marks for record book, if included, is added to practical/ clinical.
Practical/ Clinical internal assessment calculation is
Practical/ Clinical 40 marks + Viva 10 marks = Total 50 marks

12. Minimum eligibility criteria of internal assessment to appear for University


examination:
a. Theory (50) - 40%, that is 20/ 50 separately
b. Practical (50 i.e. practical 40 + viva 10) - 40%, that is 20/ 50 separately
c. Theory (50) + Practical (50) - 50 %, that is 50/100 combined
d. Theory and practical marks are separately uploaded on the University portal.

13. The hard copy of internal assessment marks of students eligible to appear for
University examination is to be signed by the concerned student, the concerned
subject HOD and the Dean of the medical college. Signed hard copy is to be
submitted to the COE 15 days before the commencement of the University Theory
examination as per Table 2.
Table 2: Internal assessment marks of students eligible for University examination

Subject:

S No. Reg No. Name Theory Practical Total Signature


Marks/ 50 Marks/ 50 Marks/ of student
100

Signature of HOD
Signature of Dean

14. The list of students not eligible to appear for the University examination is to be
submitted to the COE 15 days before the commencement of the University Theory
examination as per Table 3.
38

Table 3: Students not eligible for University Examination


University Examination:

S No. Reg No. Name of student

Signature of all HODs of concerned professional phase subjects

Signature of Dean

Internal assessment marks only of students eligible to appear for the University examination
are to be uploaded on the University portal 7 days before the commencement of the University
Theory examination.

15. Internal assessment marks once uploaded cannot be improved or changed at any
point of time, for any reason.
16. Institutions should formulate policies for remedial measures for students who are
either not able to score qualifying marks or have missed on some assessments due
to any reason(s)
17. Documents for internal assessment, including theory answer papers, log books
should be preserved by the concerned department till the candidate passes the said
professional year University examination. These documents are subject to scrutiny
by the University authorities without prior notice.

11.2. UNIVERSITY EXAMINATIONS

Summative assessment consists of University examinations. Each theory paper will have 100
marks. Marks distribution for various subjects are as follows:
39

Table 4: Marks distribution for subjects in University examinations

Phase of Course Written Total Practical/ Clinical Total


Theory
Total
Practical/ Viva
Clinical
First Professional
Anatomy - 2 papers 200 80 20 300
Physiology - 2 papers 200 80 20 300
Biochemistry - 2 papers 200 80 20 300
Second Professional
Pharmacology - 2 papers 200 80 20 300
Pathology - 2 papers 200 80 20 300
Microbiology - 2 papers 200 80 20 300
Third Professional Part – I
Forensic Medicine & Toxicology -1 100 80 20 200
paper
Ophthalmology - 1 paper 100 80 20 200
Otorhinolaryngology - 1 paper 100 80 20 200
Community Medicine - 2 papers 200 80 20 300
Third Professional Part – II
General Medicine - 2 papers 200 160 40 400
General Surgery - 2 papers 200 160 40 400
Pediatrics - 1 paper 100 80 20 200
Obstetrics & Gynaecology - 2 papers 200 160 40 400

NOTE: Internal assessment marks will not be added to University examinations marks but
will be shown separately in the grade card. At least one question in each paper of the
clinical specialties should test knowledge - competencies acquired during the
professional development programme (AETCOM module); Skills competencies
acquired during the Professional Development programme (AETCOM module) must
be tested during clinical, practical and viva.
In subjects that have two papers, the learner must secure at least 40% marks in each of the
papers with minimum 50% of marks in aggregate (both papers together) to pass in the
said subject.
Pass Criteria for University Examinations

I. Theory: Minimum 40% marks separately in Paper I & Paper II and 50% in both
papers together (Paper I + Paper II).
II. Practical/ Clinical: Minimum 50% marks (Practical + Viva)

Note: Viva marks are added only to practical & not to theory.

III. Combined Theory & Practical: Minimum 50% marks in Theory + Practical/
Clinical (including viva)
40

Table 5: Minimum eligibility marks to pass at University examination

THEORY

Paper I Paper II Total

Maximum Minimum pass Maximum Minimum Maximum Minimum


marks marks pass marks pass

100 40 100 40 200 100

PRACTICAL

Maximum Minimum pass Maximum Minimum


marks marks pass

100 50 200 100

COMBINED THEORY & PRACTICAL

Maximum Minimum pass Maximum Minimum Maximum Minimum


marks marks pass marks pass

200 100 300 150 400 200

University Examination Schedule

The regular University examinations will be held in the month of September for first &
second phase and in October for final phase part 1. The examination for final phase part 2
will be held in the month of January as follows:
Jan Feb Mar Apr May Jun Jul Aug Sep Oct Nov Dec
Foundation I MBBS
Course
I MBBS - Regular Exam I II MBBS
MBBS
September
I MBBS - Supplementary November
II MBBS - Regular Exam II III MBBS
MBBS
September
II MBBS - Supplementary November
III MBBS Part 1 - Regular Exam III Elective &
MBBS (I) Skills
October
III MBBS Part 1 - Supplementary December
III MBBS Part 2
41

Regular Supplementary Internship


Exam Exam
III MBBS III MBBS
Part 2 Part 2
January March
Internship

Supplementary Examinations:

The supplementary examinations for every Professional examination shall be scheduled a month
from the declaration of results of the regular examination.

Eligibility to appear for the supplementary examination are:

1. Students who have failed in the regular examination.


2. Students who were not eligible to write the regular exams are also eligible to write
supplementary examinations if the following conditions are fulfilled:
a. The College has submitted to the University the list of ineligible students with
reasons thereof, seven days prior to the commencement of regular examination.
b. The college has taken systematic efforts to make the ineligible students eligible by
taking extra classes, conducting additional examinations, additional practical or
clinical practices.

Colleges are required to commence the next Professional year course as per schedule irrespective
of the declaration of University results, except internship.

University Examination medal winners will be drawn on the basis of highest marks secured only
at the Regular University Examination, excluding internal assessment.

University Examination Theory Marks


The allocation of marks for University theory examinations is as follows for each paper out of a
total of 100 marks and 3 hours: (All Questions are compulsory; no choices would be given)

Part I: MCQs* 20 X 1 mark each 20 marks

Part II: Structured LAQs 3 X 10 marks each 30 marks Part

III: SAQs 10 X 5 marks each 50 marks

* MCQs are to be answered in the first 30 minutes of Examinations. NO negative marks for
wrong answer. Overwriting and correction in the MCQ answers is not permitted.
42

Example of theory paper and some examples of questions are given in Annexure 2. Model
question papers and practical examination model for each subject is provided at the end of
this document.

University Examinations

1. University examinations are to be designed with a view to ascertain whether the


candidate has acquired the necessary knowledge, minimal level of skills, ethical and
professional values with clear concepts of the fundamentals that are necessary for
him/her to function effectively and appropriately as a first contact physician. Assessment
shall be carried out on an objective basis to the extent possible.

2. Practical/ clinical examinations will be conducted in the laboratories and/ or hospital wards.
The objective will be to assess proficiency and skills to conduct experiments, interpret data
and form a logical conclusion. Clinical cases kept in the examination must be common
conditions that the learner may encounter as a physician of first contact in the community.
Selection of rare syndromes and disorders as examination cases is to be discouraged.
Emphasis should be on candidate’s capability to elicit history, demonstrate physical signs,
write a case record, analyze the case and develop a management plan.

3. Viva/ oral examination should assess approach to patient management, emergencies,


attitudinal, ethical and professional values. Candidate’s skill in interpretation of common
investigative data, X-rays, identification of specimens, ECG, etc. is also assessed.

4. There shall be one main examination in an academic year and a supplementary


examination held for all Professional examinations (MBBS I, II, III Part 1 and III Part 2)
at the earliest feasible date, scheduled after a month of declaration of results of the regular
examination and not later than 90 days after the declaration of the results of the main
examination.

5. A student shall not be entitled to graduate after 10 years of his/ her joining of the first
part of the MBBS course. Graduation includes the completion of internship, therefore, a
student is required to pass the MBBS Part II examination within 9 years of joining the I
MBBS course and complete internship in the subsequent year.

6. University Examinations shall be held as under:

(a) First Professional

1. The first Professional examination shall be held at the end of first Professional
Training (1+12 months), in the subjects of Human Anatomy, Physiology and
Biochemistry.

2. A maximum number of four permissible attempts would be available to clear the first
Professional University examination, whereby the first Professional course will have to
be cleared within 4 years of admission to the said course. Partial attendance at
43

any University examination shall be counted as an availed attempt. Registering for an


University examination is considered as an attempt irrespective of appearance for the
examination. A candidate cannot register in part or separately in individual subjects
during the first appearance at the Professional Examination.
3. Passing in first Professional is compulsory before proceeding to phase II training

(b) Second Professional


1. The second Professional examination shall be held at the end of second professional
training (11 months), in the subjects of Pathology, Microbiology, and Pharmacology. A
candidate cannot register in part or separately in individual subjects during the first
appearance at the Professional Examination.
2. Passing in second Professional examination is not compulsory before starting third
Professional Part I training; however, passing second Professional is compulsory for
being eligible for the third Professional Part I examination.

(c) Third Professional


1. Third Professional Part I shall be held at end of third Professional part 1 of training
(12 months) in the subjects of Ophthalmology, Otorhinolaryngology, Community
Medicine, Forensic Medicine and Toxicology. A candidate cannot register in part or
separately in individual subjects during the first appearance at the Professional
Examination.
2. Passing in third Professional (Part I) examination is not compulsory before
starting part II training; however, passing of third Professional (Part I) is
compulsory for being eligible for third Professional (Part II) examination.

3. Third Professional Part II - (Final Professional) examination shall be at the end


of training (14 months, including 2 months of electives) in the subjects of General
Medicine, General Surgery, Obstetrics & Gynaecology and Pediatrics. The discipline of
Orthopaedics, Anaesthesiology, Dentistry and Radiodiagnosis will constitute 25% of the
total theory marks incorporated as a separate section in paper II of General Surgery. The
discipline of Psychiatry and Dermatology, Venereology and Leprosy (DVL), Respiratory
Medicine including Tuberculosis will constitute 25% of the total theory marks in
General Medicine incorporated as a separate section in paper II of General Medicine. A
candidate cannot register in part or separately in individual subjects during the first
appearance at the Professional Examination.

Appointment of Examiners:

(a) Medical teacher appointed as an examiner in the particular subject must have at least
four years of total teaching experience as assistant professor after obtaining
postgraduate degree, MD/ MS in the subject in a college affiliated to a recognized/
approved/ permitted medical college.
(b) For the Practical/ Clinical examinations, there shall be atleast four examiners for 100
students, out of whom not less than 50% must be external examiners. Of the four
examiners, the senior-most internal examiner will act as the Chairman and co-ordinator
44

of the whole examination programme so that uniformity in the matter of assessment of


students is maintained. Where students appearing are more than 100, two additional
examiners (one external & one internal) for every additional 50, or for a part there of
students appearing, be appointed. If required number of internal examiners are not
available for Practical examination, University would postpone the Practical examination of
that college.
(c) In case of non-availability of medical teachers, approved teachers without a medical
degree (engaged in the teaching of MBBS students as whole-time teachers in a
recognized medical college), may be appointed examiners in their concerned subjects
provided they possess requisite doctorate qualifications and four years teaching
experience (as Assistant Professors of MBBS students), after having obtained a PhD in
the subject. Further, it is required that 50% of the examiners (Internal & External) are
from the medical qualification stream.
(d) External examiners may not be from the same University.
(e) The internal examiner in a subject shall not be external examiner for a college from
which external examiner is appointed in his/ her subject.
(f) A University having more than one college shall have separate sets of examiners for
each college, with internal examiners from the concerned college.
(g) External examiners shall rotate at an interval of 2 years.
(h) There shall be a Chairman of the Board of paper-setters who shall be an internal
examiner and shall moderate the questions.
(i) All eligible examiners with requisite qualifications and experience can be appointed
internal examiners by rotation in their subjects.
(j) All theory paper assessment should be done as a central assessment program (CAP) of
concerned university.
(k) Internal examiners should be appointed from same institution for unitary examination
in the same institution. For pooled examinations at one centre approved internal
examiners from the same university may be appointed.
(l) Grace marks: Grace marks up to a maximum of five marks may be awarded, at the
discretion of the University, to the total marks obtained in Paper I and Paper II theory
examinations together, only under the following conditions:

i) Student should have written all the subjects of that professional phase at that
examination and failed in only one subject of that professional phase. Grace marks are
not awarded to student who fails in more than one subject in that professional phase.
ii) Grace marks are not to be awarded for passing a subject/s resulting from exemption of
other subjects of that professional phase examination.
iii) Grace marks are only for Theory and not for Practical/Clinical examinations.

(m) All theory answer papers will be double evaluated by an internal and an external
examiner, therefore re-evaluation of papers is not permitted. Re-totalling of theory
papers is permissible.
45

Annexure 1
Phase wise schedule of internal assessment examinations

Phase Remarks
Minimum Number of tests during the year

1st ECE assessment should be included


Human Anatomy 3, Physiology
subject-wise. There should be at least
3,Biochemistry 3, Community Medicine 1 one short question from AETCOM in
each subject.

One of the 3 tests in preclinical


subjects should be prelim or pre-
university examination.

2nd Pathology 3, Pharmacology 3,Microbiology 3 Clinical subjects should also be


assessed at end of each posting
Two tests for General Medicine (Including (EOP) - Theory and Practical
Psychiatry, Dermatology, Venereology &
Leprosy (DVL) and Respiratory Medicine There should be at least one short
question from AETCOM in each
including Tuberculosis), General Surgery
subject
(Including Orthopaedics, Dentistry,
Anaesthesiology and Radiodiagnosis), One of the 3 tests in Para- clinical
Obstetrics & Gynaecology, Forensic Medicine subjects should be prelim or pre-
& Toxicology and Community Medicine university
End of posting (EOP) examination at each examination
clinical posting including those of allied
subjects

3rd Forensic Medicine & Toxicology 2, Clinical subjects should also


be tested at end of each posting
Community Medicine 2
(EOP)-Theory and Practical
Ophthalmology 2, Otorhinolaryngology 2,
There should be at least one short
Two tests for General Medicine (Including question from AETCOM in each
Psychiatry, Dermatology, Venereology & subject
Leprosy (DVL) and Respiratory
One of the tests in Ophthalmology,
Medicineincluding Tuberculosis), General
Otorhinolaryngology /Forensic
Surgery (Including
Medicine &Toxicology/
Orthopaedics,Anaesthesiology and Community Medicine should be
Radiodiagnosis), Pediatrics, Obstetrics prelim or pre-university
&Gynaecology examination
EOP examination at each clinical posting
including allied subjects
46

4th Two Tests for- Clinical subjects should also be


General Medicine (Including Psychiatry, tested at end of each posting
Dermatology, Venereology & Leprosy (DVL) (EOP) -Theory and Practical. There
and Respiratory Medicineincluding should be at leastoneshort question
Tuberculosis),General Surgery (Including from AETCOM in each subjectOne of
Orthopaedics,Anaesthesiology and the tests in Medicine,
Radiodiagnosis), Pediatrics, Obstetrics & Surgery, Pediatrics and
Gynaecology Obstetrics & Gynaecology
should be prelim or pre- university
EOP examination at each clinical posting examination
including that in allied subjects
Assessment of electives to be
included in IA AETCOM: Attitude,
Ethics and Communication

Twenty five percent of weightage in theory tests in Medicine and Surgery should be given to allied
subjects and there should be at least one question from each allied subject.
Annexure 2

Examples of theory questions

Sl.No. Type Explanation Examples


The question should pose A 6 days old term neonate has
1 Long Answer a clinical/ practical presented with jaundicenoted at 3
Question (LAQ) question problem to the days of age. He is born out of normal
students and require delivery at home. On examination, he
them to apply knowledge looks pale, has a liver of 5 cms and
and integrate it with spleen of 2 cms. Other
disciplines. Avoid giving systemicexamination is
one liners as questions. normal.
The question stem
should be structured and a.What is your provisional diagnosis?
marking distribution b.Which other conditions need to be
should be provided. Use considered?
action verbs from higher c. Enumerate the lab tests that you
domainsas given in this will order andtheir likely reports in
document.Please avoid each of the diagnosis that
simple recall based youconsidered.
questions.What is asked in d.Explain the physical findings in the
the examination generally light ofunderlying derangements.
setsthe agenda of what and
Describe the clinical
how the students learn.
features,complications and
management of type 2 diabetes
mellitus.
(3+3+4=10)
47

These provide opportunity 1. What are the various ways in


2. Short Answer to sample a wider content, which acute glomerulonephritis can
Question (SAQ) albeit in a short time. The present during childhood?
questions should be task 2. What is the role of antibiotics in
oriented rather than ‘Write childhood diarrhoeas?
a short note on xxx’. (Two 3.What is the utility of
questions based onECE in routine vitamin K
Phase 1 in internal administration during new born
assessment) (Two questions period?
based on integration in 4. Compare and contrast the use of
ramipril and amlodipine in
Phase 2 &3 in internal
treatment of hypertension.
assessment)

Reasoning These provide excellent Which components of breast milk


3. Questions opportunities for testing help inprevention of neonatal
integration, clinical infections? How do they help in
reasoning and analytic prevention of infection?
ability of the student. 2. Plan immunization for a 2 years old
totally un-
immunized child.
3. What is the physiological basis of
origin ofrespiratory sounds? How can
they help us inmaking a diagnosis?
4. Explain why adrenaline is the
preferred medication
in anaphylactic shock.

SAQ Applied (Pre & Para-Clinical Pre & Para-Clinical subjects:


4. aspects subjects: questions on Describe clinical significance Applied
applied aspect) of half-life of drugs.
(Clinical subjects: Clinical subjects: Explain patho-
questions on preclinical physiological basis of clinical
basis) features of heart failure

Short notes
5 (One question on Pharmacovigilance program of India
AETCOM
AETCOM in all subjects in AETCOM: What are the rights of a
all phases) patient in a hospital setting
MCQs
6 MCQs should be scenario A 25 year old lady was using oral
based, single response contraceptives successfully for last
with 4 options in answers. two years. She got tuberculosis and
Avoid one liner and was prescribed Rifampicin. She
negative terms in stem of became pregnant after 2months of
question. Avoid ‘all of starting Rifampicin despite
above’ and ‘none of above’in continuing the oral contraceptives.
options. Which ofthe following effects of
Rifampicin can be the reason for
this?
48

A. Induction of oral contraceptive


metabolism
B. Stimulation of ovulation
C. Interruption of entero-hepatic
circulation
D. Increased excretion of oral
contraceptives
Key: A
MCQs
2. A 2 year old child presents with
excessive weight gain over last 1
week. He has puffy eyes, pitting
edema and normal blood pressure.
Urine examination shows no RBCs
but massiveproteinuria. Which of the
following biochemical parameters is
likely to be elevated in this
child?
A. Urea
B. Cholesterol
C. Creatinine
D. Uric acid
Key B

3. Which of the following term best


describes the decreased effects of
beta adrenergic agonists in
bronchial asthma after long term
use?
A. Pharmacokinetic tolerance
B. Pharmacodynamic tolerance
C. Tachyphylaxis Drug dependence
Key: B

Note: AETCOM question should be based on competencies (primarily knowledge based)


acquired during teaching of the AETCOM module. At least one question in each paper should
test knowledge - competencies acquired during the professional development programme
(AETCOM module); Skills competencies acquired during the Professional Development
programme (AETCOM module) must be tested during clinical, practical and viva.

In subjects that have two papers, the student must secure at least 40% marks in each of the
papers with minimum 50% of marks in aggregate (both papers together) to pass in the said
subject.

--------------------------------------------------------------------------------------
49

Theory Model Question Paper and Practical Model for University Examinations

1. Anatomy
2. Physiology
3. Biochemistry
4. Pathology
5. Microbiology
6. Pharmacology
7. Forensic Medicine
8. Community Medicine
9. Ophthalmology
10. Otorhinolaryngology
11. Medicine
12. Surgery
13. Obstetrics and Gynaecology
14. Paediatrics
50

ANATOMY
THEORY QUESTION PAPER PATTERN
THEORY II PAPERS: 200 Marks (100 marks each paper)
Total Marks :100 Time : 3 hrs

1. MCQs : 20 x 1 =20 - 30 mins

2. Essay : 3 x 10 = 30 - 2 1/2hour

3. Short Answers : 10 x 5 = 50

PRACTICAL: 80 MARKS100 Marks


VIVA : 20 MARKS

THEORY QUESTION PAPER WITH REGION WISE DISTRIBUTION OF


MARKSINCLUDING MCQ
I. PAPER- 1 100 marks
1. Upper Limb & Lower Limb = 30 Marks
2. Abdomen & Pelvis = 25 Marks
3. General Embryology = 5 Marks
4. Systemic Embryology =5 Marks
5. General Histology = 5 Marks
6. Systemic Histology = 5 Marks
7. AETCOM = 5 Marks
One essay should be from upper limb, one essay from lower limb & one essay from
Abdomen & Pelvis
8. MCQ = 20 Marks
(UL5+LL5+Abd6+GE1+SE1+GH1+SH1 = 20)

II. PAPER – 2 100 marks


1. Thorax = 15 Marks
2. Neuroanatomy = 10 Marks
3. Head & Neck = 30 Marks
4. General Anatomy = 5 Marks
51

5. Genetics = 5 Marks
6. Systemic Embryology = 5 Marks
7. Systemic Histology = 5 Marks
8. AETCOM = 5 Marks
One essay should be from Thorax&Two essays from Head & Neck
9. MCQ = 20 Marks
(TH6+NA4+H&N6+GA1+GE1+SE1+SH1 = 20)

PRACTICAL & VIVA EVALUATION PATTERN WITH MARK


DISTRIBUTION
TOTAL MARKS – 100
PRACTICAL – 80 MARKS
I. GROSS ANATOMY: (40 Marks)
1. Spotters: 15 x 2 = 30
Upper Limb -2
Lower Limb -2
Thorax -2
Abdomen -2
Pelvis -2
Head & Neck -2
Neuroanatomy -2
Sectional Anatomy -1
(Specimen/Chart)

2. Discussion : 2 x 5 = 10 (Paper -1/Paper -2)

II. HISTOLOGY: (40 Marks)

1. Spotters : 10 x 2 = 20 (General Histology–4 &Systemic Histology–6)


2. Discussion : 1 x 5 = 5(1General or 1Systemic Histology slide)

3. OSPE : 3 x 5 = 15 (Genetics – 1 & Clinical Anatomy - 2)

VIVA - 20 MARKS
Osteology -5
Embryology -5
Radiology -5
Surface Anatomy -5
52

ANATOMY PAPER -1
Date:………….. Time: 10.00 AM – 01.00 PM Max. Marks: 80

• Attempt all the questions.


• Draw diagrams wherever necessary with suitable colours.

I. ESSAYS: (10X3 = 30)

1. A College student fell on his right shoulder in a motor cycle accident and it was observed
that his right arm was adducted and medially rotated and his right forearm was extended
and pronated.

a) Write the name of the clinical condition with the mention of the spinal root values
affected. Describe Brachial plexus under the following headings:
b) Formation
c) Branches & muscles supplied by the branches. (2+2+6 = 10)

2. Due torepeated childbirths, a female felt a structure coming out of vagina, especially
while straining.
a) What is the diagnosis &
b) two clinical conditions that can cause this. Write about Uterus under the following
headings:
c) Normal position
d) Supports. (1+2+2+5 = 10)

3. A traffic police came with complaints of dilated tortuous structures in his legs & swelling
of both feet on prolonged standing.

a) What is the clinical condition.


b) Write about the formation & termination of great & short saphenous veins .
c) Add a not on the perforator veins.
(1+4+5=10)

II. SHORT NOTES: (10x5 = 50)

1. Femoral Triangle 6. Histology of bone


2. Gluteus maximus muscle 7. Notochord
3. Dorsalis pedis artery 8. Development of Pancreas
4. Gall bladder 9. Histology of Liver
5. Rectus sheath 10. Ideal ways of handling of cadaver
53

ANATOMY PAPER -II


Date:………….. Time: 10.00 AM – 01.00 PM Max. Marks: 80

• Attempt all the questions.


• Draw diagrams wherever necessary with suitable colours.

I. ESSAYS: (10X3 = 30)

1. 45-year-old male, a chronic smoker & a diabetic, presented to the OPD with h/o
breathlessness & a compressive pain in the chest radiating to left shoulder tip on exertion.
Patient is relieved of pain after rest. ECG is normal.

a) What is the diagnosis for this condition.Describe the


b) origin
c) course
d) branches,
e) distribution of coronary arteries
f) What is meant by coronary dominance.
(1+1+3+4+1=10)

2. A 50-year-old lady, complained of palpitation, restlessness for a month’s duration along


with a midline neck swelling since 6 months. Examination revealed - A Nodular swelling
moving with deglutition, mild tremor of the hands, tachycardia, slightly prominent
eyeball. (1+2+1+1+2+3 =10)
a) What is the suspected clinical condition?
b) What is the embryological basis for the swelling moving on deglutition?
Describe the gross features of the involved gland under the following headings:
c) location,
d) parts,
e) coverings &
f) Histology

3. A 60-year-old female, who underwent superficial parotidectomy surgery came with


complaints of excessive sweating over the affected side of the face whenever she attempts
to eat.
a) What is the diagnosis
b) explain the anatomical basis of the lesion. Also describe the
c) external features
d) secretomotor pathway &
e) structures lying inside the Parotid gland. (1+2+3+3+1 = 10)
54

II. SHORT NOTES: (10x5 = 50)

Thoracic duct 6. Superior mediastinum – Boundaries and


contents
2. Scalp – Layers and applied aspects 7. Development of Inter atrial septum
3. Cadaveric oath 8. Circle of Willis
4. Muscles of the Pharynx and nerve 9. Sex linked recessive inheritance
Supply
5. White matter of cerebrum 10. Synovial joint
55

PHYSIOLOGY

1. SUBJECT: PHYSIOLOGY MINIMUM MARKS FOR PASSING

THEORY – 2 PAPERS 100 MARKS EACH 40 IN EACH PAPER


THEORY 2 X 100 = 200 100
PRACTICALS = 100 (VIVA=20 MARKS) 50
---------------------------------------------------------------------------------------------------------
TOTAL = 300 150
---------------------------------------------------------------------------------------------------------
PRACTICALS = 80
VIVA =20
---------------------------------------------------------------------------------------------------------
TOTAL =100 50
---------------------------------------------------------------------------------------------------------

2. ELIGIBILITY IN INTERNAL ASSESSMENT

THEORY = 50 20
PRACTICAL = 50 (VIVA = 10 IF INCLUDED) 20
---------------------------------------------------
TOTAL =100 50
----------------------------------------------------

3. UNIVERSITY THEORY DISTRIBUTION OF MARKS

MCQs 20 x 1 = 20 MARKS
LAQs 3 x 10 = 30 MARKS
SAQs 10 x 5 = 50 MARKS
56

4. DISTRIBUTION OF CONTENT OF PHYSIOLOGY DEPARTMENT

PAPER I – General Physiology , Blood, ANS, Gastro intestinal system, Endocrine Physiology,
Physiology of reproduction and Renal system, AETCOM .

PAPER I MCQs LAQs SAQs


General physiology 3x1=3 1x15=15 1x5=5
Blood 4 x1=4 2x5=5
ANS 1 x1=1 1 x5=5
Gastro intestinal system 3 x1=3 1x15=15 1 x5=5
Endocrine Physiology 3 x1=3 2 x5=5
Physiology of reproduction 3 x1=3 1 x5=5
Renal system 3 x1=3 1 x5=5
AETCOM(5) 1 x5=5
TOTAL 20 30 50

Applied questions should comprise 20 marks of each paper of 100 marks.

PAPER II –Cardiovascular system, Respiratory System& environmental system, Nerve


muscle Physiology, Neurophysiology, Special sense, Integrative Physiology + AETCOM (1
SHORT NOTE)

PAPER II MCQs LAQs SAQs


Cardiovascular system 4 x1=4 1x15=15 2x5=10
Respiratory System& 4 x1=4 2x5=10
environmental system
Nerve muscle Physiology 3 x1=3 1x5= 5
Neurophysiology 5 x1=5 1x15-15 2x5=10
Special sense 3 x1=3 1x5= 5
Integrative Physiology 1 x1=1 1x5= 5
AETCOM(5) 1x5= 5
TOTAL 20 30 50

Applied questions should comprise 20 marks of each paper of 100 marks.


57

5. UNIVERSITY PRACTICALS DISTRIBUTION OF MARKS FOR PHYSIOLOGY


DEPAERTMENT

VIVA = 20 MARKS
SUBJECT EXERCISES = 80 MARKS

Haematology : 20 marks
OSPE - Skilled : 5 marks
OSPE - Non skilled : 5 marks
Experiment : 10 marks

Clinical : 60 marks
OSCE - Skilled : 20 marks
OSCE - Non skilled : 20 marks
Major : 10 marks
2 Minor : 10 marks
Total practical marks : 80 marks
Theory Viva : 20 marks
58

MODEL QUESTION PAPER - PAPER I& PAPER II


PHYSIOLOGY
Paper - 1
Time: Three Hours Maximum Marks: 100

Illustrate your answers with suitable diagrams


( General Physiology , Blood, ANS, Gastro intestinal system, Endocrine Physiology,
Physiology of reproduction and Renal system, AETCOM )

Main question (3x 10=30)


1. a. Explain in detail about the mechanism of coagulation (7+3=10)
b. Explain fibrinolytic system

2. a. Mechanism of formation of Thyroid hormone (5+5=10)


b. Functions of Thyroid hormone

3. a. Define GFR (2+3+5=10)


b. what are the factors regulating GFR
c. Explain the regulation of GFR

3. Explain briefly: (10X5=50)


a) Explain secondary active transport. Give two examples.
b) Stages of erythropoiesis.
c) Tests for ovulation.
d) Describe the micturition reflex. Add a note on atonic bladder.
e) Mechanism of action of growth hormone.
f) Physiological actions of insulin
g) A 23 old women complains of constant pain in the upper part of the abdomen, the pain
is worse between meals, she often wakes up with severe pain, test for occult blood in
the stools were positive.
I) What is the diagnosis?
II) What is the physiological basis of treatment?
III) Why does the pain worsen between meals and at night?
h) Rh incompatibility
i) Flight and fight reaction
j) How to you demonstrate empathy in a patient encounter
59

Paper – II

Time: Three Hours Maximum Marks: 100

Illustrate your answers with suitable diagrams


(Cardiovascular system, Respiratory System& environmental system, Nerve muscle
Physiology, Neurophysiology, Special sense, Integrative Physiology + AETCOM ) Explain briefly
10x3=30
1 a. Define cardiac output (2+5+3=10)
b. Discuss in detail the factors regulating the cardiac output
c. The patient develops breathlessness on lying down and is relieved on erect posture.
Explain the pathophysiology of this condition.
2 a. Classify the descending tracts (2+5+3=10)
b. With a neat diagram, trace the corticospinal pathway and list the functions of this tract.

c. Write down the features of upper neuron lesion

3. a. With a neat diagram explain the visual pathways (5+5=10)


b. Explain the effects of lesions at various levels of visual pathway

4. Write short notes on (5x10=50)


a) Short term regulation of blood pressure.
b) Special features of coronary circulation.
c) Role of surfactant in pulmonary function. Write a note on ARDS.
d) Role of “pattern generators‟ in the regulation of respiration.
e) Acclimatization to high altitude. Add a note on chronic mountain sickness.
f) Describe the pain pathway. Add a note on pain gating.
g) Functions of hypothalamus.
h) Theory of Hearing. Add a note conductive deafness.
I) Excitation contraction coupling in skeletal muscle
J) Enumerate and describe professional qualities and roles of physician.
60

BIOCHEMISTRY

1. SUBJECT: BIOCHEMISTRY
MINIMUM MARKS FOR PASSING
THEORY – 2 PAPERS 100 MARKS EACH 40 IN EACH PAPER
THEORY 2 X 100 = 200 100
PRACTICALS = 100 (VIVA=20 MARKS) 50
TOTAL = 300 150
--------------------------------------------------------------------------------------------------------
PRACTICALS = 80
VIVA = 20
TOTAL = 100 50
---------------------------------------------------------------------------------------------------------

2. ELIGIBILITY IN INTERNAL ASSESSMENT

THEORY = 50 20
PRACTICAL = 50 20
TOTAL =100 50

3. DISTRIBUTION OF MARKS FOR THEORY PAPERS

MCQs (15 DIRECT & 5 CASE BASED) : 20 x 1 MARK = 20 MARKS


LAQs (1 CASE BASED) : 3 x 10 MARKS = 30 MARKS
SAQs : 10 x 5 MARKS = 50 MARKS

4. DISTRIBUTION OF CONTENT:

Paper-I

S. No TOPICS Competency No

Basic Biochemistry
BI 1.1
1. (Cell and its organelle)
BI 6.6
Bioenergetics & Electron transport chain
2. Enzymes BI 2.1 – BI 2.7

3. Chemistry and Metabolism of Carbohydrates BI 3.1 – 3.10


Vitamins BI 6.5
4.
Mineral metabolism BI 6.9 - BI 6.10
Chemistry and metabolism of nucleotides BI 6.2 – BI 6.4
5.
Molecular biology BI 7.1 – BI 7.4
Xenobiotics BI 7.5
6.
Free radicals and anti-oxidants BI 7.6 - BI 7.7
Organ function test – kidney, adrenal , liver and thyroid function
7. BI 6.13 – BI 6.15
tests
61

Paper-II

S. No TOPICS Competency No

Chemistry and Metabolism of Lipids BI 4.1 – BI 4.7


1.
Metabolism of fed and fasting BI 6.1

2. Chemistry and Metabolism of proteins BI 5.1 – BI 5.5

3. Water, electrolyte and acid base homeostasis BI 6.7 - BI 6.8

BI 5.2,
Chemistry and Metabolism of Haemoglobin
4. BI 6.11, BI 6.12
Extra cellular matrix
BI 9.1 – BI 9.3
5. Nutrition BI 8.1 – BI 8.5
Oncology BI 10.1 – BI 10.2
6.
Immunology BI 10.3 – BI 10.5

7. Instrumentation BI 11.19

5. MARK DISTRIBUTION

PAPER – I

No. of No. of No. of


S. No TOPICS Competency
MCQ’s LAQ SAQ
No
Basic Biochemistry
(Cell and its organelle) BI 1.1
1. 2 - 2
Bioenergetics & Electron BI 6.6
transport chain
2. Enzymes BI 2.1 – BI 2.7 2 - 1
Chemistry and Metabolism of
3. BI 3.1 – 3.10 4 1 1
Carbohydrates
Vitamins BI 6.5
4. 4 1 2
Mineral metabolism BI 6.9 - BI 6.10
Chemistry and metabolism of
BI 6.2 – BI 6.4
5. nucleotides 4 1 1
BI 7.1 – BI 7.4
Molecular biology
Xenobiotics BI 7.5
6. 2 - 1
Free radicals and anti-oxidants BI 7.6 - BI 7.7
Organ function test – kidney,
BI 6.13 – BI
7. adrenal , liver and thyroid function 2 - 1
6.15
tests
8. AETCOM - - 1
Total marks 20 30 50
62

PAPER II
Competency No. of No. of No. of
S. No TOPICS
Number MCQ’s LAQ SAQ
Chemistry and Metabolism of Lipids BI 4.1 – BI 4.7
1. 4 1 2
Metabolism of fed and fasting BI 6.1

2. Chemistry and Metabolism of proteins BI 5.1 – BI 5.5 4 1 1

Water, electrolyte and acid base


3. BI 6.7 - BI 6.8 2 -
Homeostasis
1
Chemistry and Metabolism of BI 5.2,
4. Haemoglobin BI 6.11, BI 6.12 4 2
Extra cellular matrix BI 9.1 – BI 9.3
5. Nutrition BI 8.1 – BI 8.5 3 1
BI 10.1 – BI
Oncology 10.2
6. 2 - 2
Immunology BI 10.3 – BI
10.5

7. Instrumentation BI 11.19 1 - 1

8. AETCOM - - 1

Total 20 30 50

6. DISTRIBUTION OF MARKS FOR UNIVERSITY PRACTICALS

A) VIVA = 20 MARKS
B) PRACTICALS (OSPE pattern) = 80 MARKS
TEN (4 PERFORMANCE & 6 RESPONSE) STATIONS WITH 8 MARKS EACH

PERFORMANCE STATIONS: 4 nos


1: Normal component of urine
2: Abnormal component of urine
3: Qualitative detection of biomolecules
4: Quantitative detection of biomolecules

RESPONSE STATIONS: 6 nos


4 Case/Chart based Scenarios
2 Instruments & techniques

----------X----------
63

BIOCHEMISTRY –Paper I
Time: 3 hours Maximum: 100 Marks
MCQs 20 marks
Answer ALL questions
I. Long Answer Questions: (3 x 10 = 30)

1. A 3 year old orphan was admitted with guiddiness and blurring of vision. His random
serum glucose was 40 mg/dl.
a) Mention the substrates needed for glucose synthesis in fasting state.
b) Explain the gluconeogenesis pathway with its regulation. (2+8=10)

2. A 70 year old woman presented with muscle twitching and tetany. Her serum calcium was
7.8 mg/dl.
a) Write the normal serum calcium levels.
b) Explain how serum calcium is regulated in humans.
c) What are the causes for hypocalcemia? (1+6+3=10)

3. A chronic alcoholic person was brought with complaints of edema of the legs, walking
difficulty and swelling of abdomen. Ultrasound abdomen shows ascitis with no
organomegaly.
a) Mention the probable vitamin deficient in this patient
b) Write the sources and functions of this vitamin
c) How will you confirm the diagnosis in laboratory? (2+6+2=10)

II. Short Answer Questions: (10 x 5 = 50)


4. Describe the cell membrane transporters with examples. Mention the inhibitor of Na-k
ATPase pump.
5. Explain the phase-II detoxification process in humans with suitable examples.
6. Classify enzymes with examples.
7. Describe the organisation of electron transport chain complexes with their functions.
Mention their inhibitors.
8. Discuss the steps and applications of polymerase chain reaction.
9. 50 years old female complained severe pain in right great toe. Her serum uric acid was
raised. Mention the normal serum uric acid levels. How uric acid is synthesised in
humans?
10. Classify renal function tests. Describe clearance tests.
11. Write notes on anti-oxidant defence systems in humans.
12. Mention the various DNA repair mechanisms and describe any one in detail.
13. Write the rights and responsibilities of patients.
64

BIOCHEMISTRY Paper 2

Time: 3 hours Maximum: 100 Marks


MCQs 20 marks
Answer ALL questions
I. Long Answer Questions: (3 x 10 = 30)
1. A 50 years old male admitted with complaints of jaundice, altered conscious levels and
sleepless nights. He had previous history of admissions for alcoholic liver disease.
Presently he is diagnosed to have alcoholic hepatic encephalopathy due to hyper-
ammonemia.

a) How ammonia is synthesised and transported in humans


b) Explain the steps involved in ammonia detoxification in liver.
c) Enumerate the inborn errors related to ammonia detoxification. (2+6+2=10)

2. A 60 years old known diabetic came with chest pain. ECG confirmed the cardiac
involvement. Serum showed hypercholesterolemia and hyper triglyceridemia.

a) Mention the normal cholesterol and triglyceride levels


b) Explain how dietary triglycerides are transported in humans
c) Enumerate the various types of hyper-lipoproteinemias with their
molecular defects (2+5+3=10)

3. A known diabetic had semiconscious spells with fruity smell in his breath. His serum
glucose was 400 mg/dl and urine Rothera's test was positive.

a) Write the acid base imbalance expected in this patient.


b) How pH is regulated by Kidney?
c) Comment on anion gap in this condition. (2+6+2=10)
II. Short Answer Questions: (10 x 5 = 50)

4. What is the enzyme deficiency in acute intermittent porphyria? Mention its clinical
features and diagnostic tests.
5. Describe the synthesis of collagen.
6. Write in detail about how insulin regulates metabolism of nutrients in well-fed state.
7. Discuss the role of T-helper cells in immune response.
8. What are oncogenes and anti-oncogenes? How oncogenes are activated?
9. Enumerate the types and discuss the applications of electrophoresis.
10. Discuss the influence of social and economical dimensions of an illness in a patient.
11. Mention the types, causes, clinical effects and diagnostic tests of protein energy
malnutrition.
12. What is the molecular defect in HbS? Describe the sickling process, clinical features and
tests to diagnose the case of sickle cell anemia.
13. Discuss the secondary structure of proteins. Mention the structural changes seen in
Prion disease.
-------------------------------------------------------
65

PHARMACOLOGY

1. SUBJECT: PHARMACOLOGY MINIMUM MARKS FOR PASSING


THEORY – 2 PAPERS - 100 MARKS EACH 40 IN EACH PAPER
THEORY 2 X 100 = 200 100
PRACTICALS = 100 (VIVA=20 MARKS) 50

TOTAL = 300 150

PRACTICALS = 80
VIVA
= 20

TOTAL =100 50

2. ELIGIBILITY IN INTERNAL ASSESSMENT


20
THEORY = 50
PRACTICAL = 50 (VIVA = 10 IF INCLUDED) 20

TOTAL =100 50

3. UNIVERSITY THEORY DISTRIBUTION OF MARKS

MCQs 20 x 1 = 20 MARKS
LAQs 3 x 10 = 30 MARKS
SAQs 10 x 5 = 50 MARKS

4. DISTRIBUTION OF CONTENT:

Column 1 Column 2 Column 3 Column 4


Topics MCQs LAQs (3x10=30) SAQs(10x5=50)
(20x1=20marks) S1 S2 S3 S4 S5 S1 S2 S3 S4 S5
GP 4 0 1 0 1 0 2 1 2 1 3
ANS 4 1 0 1 0 1 1 3 1 3 1
CVS 4 1 1 1 1 1 1 1 1 1 1
BLOOD 4 0 0 1 0 0 3 2 1 2 2
AUTOCOIDS 2 1 0 0 0 1 0 2 2 2 0
RS 2 0 1 0 1 0 2 0 2 0 2
AETCOM 0 0 0 0 0 0 1 1 1 1 1

PAPER-I (100 marks)

I. Multiple choice questions (MCQs) 20 x 1 = 20 MARKS


II. Long answer questions (LAQs) 3 x 10 = 30 MARKS
III. Short answer questions (SAQs) 10 x 5 = 50 MARKS
66

PAPER-II (100 marks)


Column 1 Column 2 Column 3 Column 4
LAQs SAQs
Topics MCQs (3x10=30)
S1 S2 S3 S1 S2 S3
CHEMO 5 1 1 1 3 3 3
CNS 5 1 1 1 2 2 2
HORMONES 5 0 1 0 3 1 3
GIT 3 1 0 1 0 2 0
MISCELLANEOUS 2 0 0 0 1 1 1
AETCOM 0 0 0 0 1 1 1

• The above tables contain the mark distribution in various topics in Pharmacology Paper 1
& Paper 2 respectively
• The first column represents the topics in pharmacology
• The second column depicts the number of MCQs from each topic with 1 mark for one
MCQ, totally 20 MCQs per paper
• The third and fourth columns represent the long answer questions (LAQs) and short
answer questions (SAQs) with different sets (S1, S2, ...... ) in each respectively.
• Correlate S1 LAQs with S1 SAQs for equal mark distribution (e.g. if three LAQs were
asked from 3 topics, then SAQs are to be set in such a way to allocate appropriate
weightage across the topics)
• E.g: in Set 1 (S1) of paper 1, one LAQ is from ANS, one LAQ from CVS and one LAQ from
autocoids hence the SAQs are to be set to give appropriate weightage across all topics.
• Similarly, correlate Set 2 (S2) LAQ with Set 2 (S2) SAQ and so on...
• The questions in LAQs and SAQs may better be structured (indicating the allotted marks
for each sub-question) for more objectivity. (Refer Annexure 2 of Module MCI Competency
Based Assessment)

5. UNIVERSITY PRACTICALS DISTRIBUTION OF MARKS (100 MARKS)

1. PRACTICAL EXERCISES = 80 MARKS

PRACTICALS –
PART A. Spotters (10 marks)
Sl No. Topics Marks
1. Spotters * 10
67

PART-B Clinical Pharmacology (40 marks)


Sl.No Topics Marks
1 Prescription writing * 10
2 Prescription audit (CCR) * 10
3 Problem Solving Exercise 1x5=5
4 Adverse Drug Reactions (ADR) 1x5=5
5 Pharmacoeconomics 1x5=5
6 Dosage calculation 1x5=5
* The Number in each may be decided by institution

PART-C: Assessment of demonstration and communication skills #


(30 marks)
Sl.No Topics Marks
CAL (Computer Assisted Learning) -
1 Experimental Pharmacology 10
PDC (Patient Doctor Communication) -
2 AETCOM 10
Routes of drug administration(Inhalational,
3 Injections)-OSPE 10

# Table viva optional

2. VIVA VOCE = 20 MARKS

1. PRACTICAL EXERCISES = 80 MARKS


2. VIVA VOCE = 20 MARKS
TOTAL (1+2) = 100 MARKS
6. MODEL QUESTION PAPER - PAPER I& PAPER II (Set 1
was selected for both paper I and paper II)
68

PHARMACOLOGY- Paper I

Time: 3 hours Maximum marks: 100


Direction: Answer ALL Questions.
Illustrate your answers with suitable diagrams

GENERAL PHARMACOLOGY, AUTONOMIC NERVOUS SYSTEM, CARDIOVASCULAR


SYSTEM, BLOOD,AUTACOIDS, RESPIRATORY SYSTEMAND AETCOM
I. MCQs (20 x 1=20 marks)
II. Long answer questions (3 x 10=30 marks)

1. A 48-year-old male came to the casualty with complaints of retrosternal chest


discomfortradiating to the left shoulder following a bout of exercise. The pain was relieved
on taking rest. He was diagnosed as a case of classical angina.
(1+3+4+2)
a) Write one drug that can be used to abort an acute attack of this condition.
b) Explain the mechanism of pain relief for the said drug.
c) List the adverse effects, other clinical indications and precautions to be taken for the
said drug.
d) With one example each, enumerate any two drug classes for the prevention of further
attacks

2. A 30-year-old male was brought to the casualty with h/o consumption of an insecticide 30
min earlier. He presented with cough, soiled clothes due to urination and defecation. On
examination, he haddrooling of saliva, profuse sweating, miosis, low volume pulsewithBP
(90/60 mm Hg). He was diagnosed as a case of Organophosphorus poisoning (OP).
(2+1+2+5)
a) Mention the pharmacotherapy along with the rationale for the drugs chosen to treat
the above case.
b) Mention the clinical importance of “Enzyme Aging” in OP poisoning.
c) Enlist two reversible anticholinesterase drugs and their clinical indications.
d) List the adverse effects and other clinical indications of the antidote that specifically
antagonises the symptoms and signs of the above condition.

3. A 44-year-old women presented to OPD with the complaints of phalangeal joint swelling
and stiffness for the past 2 months. Clinical examination and serological investigations
confirmed as a case of Rheumatoid arthritis (2+4+2+2)
a. List four DMARD agents that may be useful in the above said condition.
b. Write briefly on the mechanism of action and adverse effects of any one non biological
agent.
c. Enlist two anti-inflammatory drugs for immediate pain relief with minimal GI adverse
effects.
d. Brief the role of corticosteroids in the above said condition.
69

III. Short Answer Questions (10 x 5 = 50)

4. With suitable examples,state the clinical significance of ‘Plasma protein binding’ in


relation to drug-drug interaction.
5. Differentiate type A and type B adverse drug reactions. Outline the pharmacological
management of each type. (2+3)
6. Enumerate two drug groups with examples that arrest/reverse the disease progression in
congestive cardiac failure. Briefly explain the rationale for their use and two adverse
effects for each. (1+2+2)
7. List two cardio selective and non-selective beta-blockers. Compare and contrast the two
groups. (2+3)
8. Briefly describe the pharmacology of any one group of drugs used in the management of
dyslipidemia.
9. Mention the stepwise management of bronchial asthma based on severity. Briefly explain
the mechanism of aspirin induced bronchial asthma. (3+2)
10. Enumerate two oral and parenteral formulations used for the management of microcytic
hypochromic anaemia due to nutritional deficiency. List two clinical situations where
parenteral formulations are indicated in the above said anaemia. Mention the precautions
to be taken prior to the administration of the said parenteral formulation. (2+2+1)
11. Enumerate two fibrinolytic drugsand their adverse effects. Briefly explain the rationale
behind the usage of fibrinolytics in acute myocardial infarction. (1+1+3)
12. Briefly explain the symptomatic drug treatment approach with their mechanism of action
involved in productive and non-productive cough.
13. A 30-year-old (Gravida-2, Para-0, Live birth-0, Abortion-1) on anti-hypertensive treatment
since last three years, with gestational age of 32 weeks came for routine antenatal check-
up. Ultrasound scan revealed that the foetus had no cardiac activity, suggestive of
intrauterine death (IUD). On detailed history it was found that she was on treatment with
Tab. ENALAPRIL 10 mg BD since three years and the BP was well controlled as per the
case records. The husband has a doubt whether the earlier abortion and the present loss
of foetus could be due to any familial reason from his wife’s side. (1+1+1+1+1)
a. Does the patient have the right to know about the probable cause of IUD?
b. Explain, if there is any maleficence on part of the drug treatment in relation to earlier
abortion and the present foetal loss?
c. If this IUD is distinguished as an adverse event, what is your professional
responsibility to a related National Program?
d. What are the shared responsibilities on the part of the patient and the physician?
e. How will you clear the husband’sdoubt empathetically?
70

PHARMACOLOGY- Paper II

Time: 3 hours Maximum marks: 100


Direction: Answer ALL Questions.
Illustrate your answers with suitable diagrams

CHEMOTHERAPY, CENTRAL NERVOUS SYSTEM, ENDOCRINE, GIT,


MISCELLANEOUS AND AETCOM

I. MCQs (20 x 1 = 20 marks)


II. Long answer questions (3 x 10 = 30 marks)

1. A female patient aged 26 yrs has come to the out-patient department with one episode of
generalized tonic–clonic seizure lasting for two minutes. Her liver functions are normal.
She was married 6 months earlier. (1+3+6= 10)
a. Enlist the drugs which can be used in this condition.
b. Explain the mechanism of action, adverse effects, and other clinical uses of any one
drug used in this patient.
c. With justification, enumerate the drugs that are contraindicated in this patient in case
she is pregnant.

2. A 40-year-old patient, weighing 50 kg,came with h/o cough and expectoration, evening rise
of temperature and loss of weight since 15days. Sputum examination and chest x-ray
confirmed the diagnosis as pulmonary tuberculosis (TB). (8+2)
a. List the drugs that are to be administered for this newly diagnosed condition and
describe the mechanism of action, adverse effects and drug interactions of any one
drug.
b. Assuming this patient to be a case of drug sensitive TB, write down the recommended
RNTCP regimen?

3. List the various targets of drugs used in the treatment of acid peptic disease with suitable
diagrams. Discuss the MOA, uses and adverse effects of any one commonly used group.
(4+6)
III. Short Answer Questions (10 x 5 = 50)

4. Name any two anti-HIV regimens and explain the rationale of using fixed drug regimen.
(3+2)
5. Mention any two hormonal drugs with their indication in cancer chemotherapy along with
the rationale for each. (2+3)
6. List any two non-benzodiazepine drugs used as anti-anxiety agents and mention the
advantages of them over benzodiazepines. (1+4)
7. Enumerate any two oral contraceptive options available for a 26-year-old woman. Briefly
describe the mechanism of action, two adverse effects and two non-contraceptive benefits
of any one of the options mentioned. (1+2+1+1)
71

8. What are the advantages of using oxytocin to induce labour? Mention any two drugs and
their mechanism of action used to control postpartum haemorrhage? (2+3)
9. Briefly describe the drug therapy of hyperprolactinemia? Mention two drugs that cause
hyperprolactinemia? (3+2)
10. With suitable examples, list two drug classes used in the management of idiopathic
parkinsonism with two adverse effects for each example. Briefly explain the rationale for
two drug combinations used. (3+2)
11. Enumerate one oral and parenterally administered cephalosporin.Add a note on third
generation cephalosporins. (2+3)
12. Enumerate any four drugs used in heavy metal poisoning. Outline the management of
acute iron poisoning with a note on the mechanism of action of the antidote. (2+2+1)
13. What is the role of non-maleficence as a guiding principle in prescribing practice?

-------------------------------------------------------------------------------------------------
72

PATHOLOGY

Subject: PATHOLOGY Minimum marks for passing

Theory: 2 papers 100 marks each 40 in each paper

Theory: 2x100 = 200 100

Practicals = 100 (Viva =20 marks) 50

Total = 300 150

Practical = 80

Viva = 20

Total = 100 50

Eligibility in Internal Assessment:


Theory = 50 20

Practical = 50 (Viva=10) 20

Total = 100 50

University Theory Distribution of Marks:


MCQ’s 20x1 = 20 Marks

LAQ’s 3x10 = 30 Marks

SAQ’s 10x5 = 50 Marks


73

Distribution of Content:
Paper I:
Paper I MCQ’s LAQ’s SAQ’s
General Pathology(50) 10 2 4
Hematology (30) 5 1 3
Clinical Pathology and Blood 5 - 2
banking (15)
AETCOM (5) - - 1
TOTAL 20 30 50

Paper I: General Pathology including Hematology and Clinical Pathology:

1. General Pathology = 50 Marks


2. Hematology = 30 Marks
3. Clinical Pathology + Blood Banking = 15 Marks
4. AETCOM = 5 Marks
Paper II:
Paper II MCQ’s LAQ’s SAQ’s
CVS, RS, GIT, HBS, Pancreas, 10 2 4
LN and Spleen ,Urinary (50)
MGT, FGT, Breast, Endocrine, 10 1 5
Musculoskeletal, Nervous
system, Skin, Eye (45)
AETCOM (5) - - 1
TOTAL 20 30 50

Paper II:
• Cardiovascular System
• Respiratory System
• Alimentary System including disease of Liver,
o Gall bladder and exocrine Pancreas, spleen ,LN = 50 marks
• Urinary System
• Endocrine System- Diseases of Thyroid, Adrenals,
o Parathyroid, Pituitary and endocrine Pancreas

• Male Genital System, Female Genital System = 45 marks


Breast, Skin, Nervous System, Eye

• Musculoskeletal system
• ETCOM = 5 marks
74

UNIVERSITY PRACTICAL DISTRIBUTION OF MARKS:

Viva = 20 marks (Gen Path+ Hematology + Systemic I + Systemic II: 5


marks each)
Practicals =80 marks

1. Problem solving exercises (8 cases, 5 marks each ) = 40 marks


(Clinical case scenarios with slides, specimens, instruments and charts)
2. Urine examination with clinical history : = 10 marks
Findings and interpretation
3. Peripheral Smear: Stained smear is given = 10 marks
Reporting and Interpretation
4. Hemoglobin estimation = 5 marks
5. Blood grouping = 5 marks
6. Histopathology slide with Clinical history: = 10 marks
Reporting and Discussion
75

Pondicherry University
Model Question Paper- Pathology- Paper I
Duration: 3 hours (including first 30 minutes for MCQ’s) Max. Marks: 100
Instructions:
• MCQ sheet will be collected at the end of 30 minutes.
• All Questions are compulsory.
• Draw neat and labeled diagrams, wherever necessary.

LAQ 1: Define neoplasia. Explain the molecular basis of carcinogenesis. Write in brief about
the Viral Carcinogenesis with suitable examples.
(1+5+4=10 marks)
LAQ 2: Describe the process of cutaneous wound healing. Add a note on factors affecting the
wound healing. What are the complications of wound healing?
(4+3+3=10 marks)
LAQ 3: A 65-year old man presented with complaints of excruciating lower backache which
had progressively worsened over the last 6 months. He also gave history of recurrent
episodes of fever over the last 3 months. The X-ray of vertebral column revealed
multiple punched-out lytic lesions involving the lumbar and sacral vertebral bodies.
ESR was high and one of the findings on his peripheral blood film was RBC rouleux
formation.

a) What is the probable diagnosis and why?


b) Explain the Pathogenesis of the disease.
c) Describe in detail the laboratory findings in this disease.
d) Add a note on biochemistry tests which are useful in this disease.
(2+3+5=10 marks)
SAQ: (10 x 5=50 marks)

4. Classify and differentiate the types of Pathological calcification with examples. (2+3 )
5. Write the etiopathogenesis of Granulomatous Inflammation .Give examples (3+2)
6. Down Syndrome- Etiology, Clinical features and Lab tests (2+2+1)
7. Pathogenesis of Type I hypersensitivity reaction with examples (3+2)
8. Haemophilia A –Pathogenesis and Lab findings (2+3)
9. Laboratory findings in Megaloblastic Anemia
10. Chronic Myeloid Leukemia –Pathogenesis and Lab findings (2+3)
11. Erythrocyte Sedimentation Rate (ESR) –Procedure and its significance (2+3)
12. Coomb’s test –Principle and applications (3 +2)
13. AETCOM Qn: How do you proceed in a case of Goitre who has come for FNAC
76

Pondicherry University

Model Question Paper- Pathology- Paper II

Duration: 3 hours (including first 30 minutes for MCQ’s) Max. Marks: 100

Instructions:
• MCQ sheet will be collected at the end of 30 minutes.
• All Questions are compulsory.
• Draw neat and labeled diagrams, wherever necessary.

LAQ 1: A 55 year old male, chronic smoker, presented with complaints of cough, hemoptysis
and weight loss. Physical examination showed signs of Cushing’s syndrome.
Radiological examination revealed a mass lesion in the lung. CT guided biopsy showed
small cells with scant cytoplasm, fine granular chromatin with nuclear moulding.

a) What is your probable diagnosis and why?


b) Write the etiopathogenesis of the disease.
c) Write about the morphology of the affected organ.
d) Add a note on diagnostic investigations you will carry out in this condition.
(2+2+3+3=10 marks)

LAQ 2: A 35 year old chronic alcoholic male presented with anorexia, weakness, occasional
hematemesis, mild jaundice and ascitis. Ultrasound abdomen showed splenomegaly
and diffuse nodularity of the liver.
a) What is your probable diagnosis and why?
b) Describe the etiopathogenesis of this condition
c) Describe the morphological changes in the liver
d) What are the complications of this disease?
(2+3+3+2 =10 marks)

LAQ 3: A 55 Year old mother of 5 children presented with complaints of post-menopausal


bleeding. Per speculum examination showed Ulceroproliferative lesion involving
cervix that bleeds on touch.

a) What is the probable diagnosis and why?


b) Describe etiopathogenesis of the disease
c) Write the morphological findings in this condition.
(2+4+4=10 Marks)
77

SAQ: (10 x 5=50 marks)

4. Morphology of heart in Rheumatic Heart Disease


5. Gall stones – Etiopathogenesis and complications (3+2)
6. Etiopathogenesis of Colorectal carcinoma
7. Minimal Change Disease –Etiopathogenesis and morphology (3+2)
8. Seminoma
9. Papillary Carcinoma Thyroid
10. Osteosarcoma - Etiopathogenesis and morphology (3+2)
11. CSF findings in Tubercular meningitis
12. Meningioma
13. AETCOM Qn: How do you proceed with investigations in a male infertility case

-----------------------------------------------------------------------------------
78

MICROBIOLOGY

1. SUBJECT: MICROBIOLOGY MINIMUM MARKS FOR PASSING

THEORY – 2 PAPERS 100 MARKS EACH 40 IN EACH PAPER


THEORY 2 X 100 = 200 100
PRACTICALS = 100 (VIVA=20 MARKS) 50
-------------------------------------------------------------------------------------------------------------
TOTAL = 300 150
-------------------------------------------------------------------------------------------------------------
PRACTICALS = 80
VIVA =20
-------------------------------------------------------------------------------------------------------------
TOTAL =100 50
-------------------------------------------------------------------------------------------------------------

2. ELIGIBILITY IN INTERNAL ASSESSMENT MINIMUM MARKS FOR PASSING

THEORY = 50 20
PRACTICAL* = 50 20
-------------------------------------------------------------------------------------------------------------
TOTAL =100 50
-------------------------------------------------------------------------------------------------------------
*VIVA = 10 (IF INCLUDED), RECORD BOOK = 5 AS IN THE GMER

3. UNIVERSITY THEORY DISTRIBUTION OF MARKS

Multiple Choice Questions (MCQs) 20 x 1 = 20 MARKS


Long Answer Questions (LAQs) 3 x 10 = 30 MARKS
Short Answer Questions (SAQs) 10 x 5 = 50 MARKS
-------------------------------------------------------------------------------------------------------------
TOTAL =100 MARKS
-------------------------------------------------------------------------------------------------------------
4. DISTRIBUTION OF CONTENT:

PAPER I - GENERAL MICROBIOLOGY, IMMUNOLOGY, CVS& BLOOD, RESPIRATORY


TRACT INFECTIONS (RTI), ZOONOTIC & MISCELLANEOUS (Z&M), AETCOM
79

PAPER I MCQs LAQs SAQs


GENERAL 5 10 10
MICROBIOLOGY(25)
IMMUNOLOGY(25) 5 10 10
CVS& B, RTI, Z&M (45) 10 10 25
• BACTEROLOGY (20) • BACTERIOLOGY (10) • BACTERIOLOGY (10)
• MYCOLOGY (5) • MYCOLOGY (5)
• VIROLOGY (5) • VIROLOGY (5)
• PARASITOLOGY (5) • PARASITOLOGY (5)
AETCOM(5) --- --- 5
TOTAL (100) 20 30 50

PAPER II – GASTROINTESTINAL AND HEPATOBILIARY (GI&HB),SKIN AND SOFT


TISSUE (SST), CNS,GENITO-URINARY& SEXUALLY TRANSMITTED INFECTIONS
(GU&STI),AETCOM

PAPER II MCQs LAQs SAQs

GI&HB, SST, CNS, 20 30 45


GU&STI
• BACTERIOLOGY (10) • BACTERIOLOGY (10)
• BACTERIOLOGY (20) • VIROLOGY (10) • MYCOLOGY (15)
• MYCOLOGY (15) • PARASITOLOGY (10) • VIROLOGY (10)
• VIROLOGY (20) • PARASITOLOGY (10)
• PARASITOLOGY (20)
AETCOM(5) --- --- 5

TOTAL (100) 20 30 50

5. GUIDELINES TO INCLUDE ORGANISMS UNDER EACH COMPETENCY IN


SYSTEM WISE COMPETENCIES AS PER MCI
80

PAPER I
GENERAL MICROBIOLOGY, IMMUNOLOGY, CVS & BLOOD, RESPIRATORY TRACT
INFECTIONS (RTI), ZOONOTIC & MISCELLANEOUS (Z&M)

GENERAL MICROBIOLOGY - MI 1.1 to 1.6


History, General concepts & Scope of Microbiology
Contributors in Microbiology & their contributions Concept
of microorganisms causing infections Sterilization and
Disinfection

Introduction to Bacteriology
Eukaryotic cell & Prokaryotic cell: Structure & functions Bacterial
morphology, physiology
Taxonomy and classification
Host bacterial interactions - Definition and function of bacterial virulence factors, toxins, enzymes
Overview of bacterial infection specimen collection and laboratory diagnosis

Introduction to Virology
Taxonomy and classification of viruses
Viral morphology, replication and virion function Host viral
interactions
Overview of viral infection specimen collection and laboratory diagnosis Introduction to
Prions

Introduction to Mycology
Taxonomy and classification of fungus and fungal diseases Morphology
of fungi and reproduction
Host fungi interactions
Overview of diagnosis of fungal infections, specimen collection and laboratory diagnosis
Introduction to Protozoology and Helminthology Taxonomy and
classification of protozoans and helminthes Concept of life cycles
Host parasitic interactions
Overview of parasitic infections, and laboratory diagnosis
Microbial genetics, Antimicrobial agents & resistance mechanisms Epidemiology
of infectious diseases
Host, parasite and environment
IMMUNOLOGY – MI 1.7 TO 1.11

Immunity
Antigens & Vaccines
Antibodies Complement
system
81

Organ & Cells of the Immune System The


Normal Immune Response Hypersensitivity
Immunodeficiency & Auto immunity
Transplantation &Tumor Immunology

CVS& BLOOD STREAM INFECTIONS – MI 2.1 TO 2.7

Rheumatic fever – Streptococcus pyogenes


Endocarditis - Streptococcus viridans, Enterococci, HACEK
Viral haemorrhagic fevers - Dengue, Chickungunya, Kyasanur Forest Disease Malaria,
Filaria, Kala azar
HIV
Microbial agents causing anaemia

RESPIRATORY TRACT INFECTIONS – MI 6.1 TO 6.3 URI:


Cdithphtheriae,
Streptococcus spp
Staphylococcus spp
Influenza virus, Adenovirus, Coronavirus, Rhinovirus Epstein
Barr Virus

LRI:
TB-Mycobacterium Tuberculosis & NTM
S pneumoniae, H influenzae
Atypical Pneumonia- Mycoplasma, Chlamydia (psittacosis and pneumoniae) Legionella
Bordatella pertussis
Influenza virus, Parainfluenza, RSV
Aspergillusspp, Pneumocystis

ZOONOTIC DISEASES AND MISCELLANEOUS - MI 8.1 TO 8.16

Zoonotic:
Anthrax
Brucellosis
Plague
Rickettsial infections - Scrub typhus
Q fever, B burgdorferi
Leptospirosis
Misc. zoonosis: Relapsing fever, BovineTB & Cat scratch disease

Opportunistic infection:
Histoplasmacapsulatum
Penicillium
Aspergillusspp
Cryptococcus neoformans
Pneumocystis jirovecii
82

Cryptosporidium
Isospora belli
Cyclospora
Microsporidia
Toxoplasma gondii
Cytomegalovirus
Mycobacterium tuberculosis
MOTT complex
Legionella pneumophila
HHV-8
Human polyomavirus 2, (JC virus)

Emerging infection:
Ebola virus, Zika virus, Nipah virus, SARS virus, MERS virus, CoViD 2019

HAI- Types & Prevention


Pseudomonas spp, Acinetobacterspp, MDROs

Oncogenic viruses
Bacteriology of milk, water and air PUO
Perinatal infections
All National health programmes are to be included with respective infectious disease.
83

PAPER II
GASTROINTESTINAL AND HEPATOBILIARY (GI&HB), SKIN AND SOFT TISSUE (SST),
CNS, GENITO-URINARY & SEXUALLY TRANSMITTED INFECTIONS (GU&STI)

GASTROINTESTINAL AND HEPATOBILIARY SYSTEM – MI 3.1 TO 3.8

Diarrhoea:
Cholera
Enteric fever, Non typhoidal Salmonellosis DiarrheagenicE coli and
Antibioitic associated diarrhea
Viral gastroenteritis – Rotavirus, Norovirus, Calcivrus, Astro virus
Enteroviruses
Giardia
Cryptosporidium
Isospora Cyclospora
Microsporidia

Dysentry:
Shigella, Campylobacter,Vparahemolyticus E
histolytica
Balantidium coli

Intestinal parasites:
Nematodes – Ascaris, Enterobius, Trichuris, Hookworm, Strongyloides

Trematodes - Fasciola
Cestodes – Taenia, Echinococcus, Diphyllobothrium

Food poisoning:
Bacillus cereus
Clostridium botulinum
Staphylococcus aureus
Mycotoxins

Acid peptic disease:


H pylori

Viral Hepatitis:
HAV, HEV
HBV, HDV, HCV
Yellow fever, CMV
Other causes of hepatitis - Leptospira
84

MUSCULOSKELETAL SYSTEM - SKIN AND SOFT TISSUE INFECTIONS – MI 4.1 TO 4.3


Staphylococcus, Streptococcus pyogenes Clostridium
perfringens, Clostridium tetani Non sporing
anaerobes - Bacteroides Melioidosis
Leprosy
Exanthematousviral diseases: Measles, Mumps, Rubella, Varicella zoster, HSV,Parvo
virus, HHV 6, HHV 7, HHV 8, Papova viruses, Pox virus Tissue nematodes-
Dracunculus, Cysticercosis, Trichinella
Superficial fungal infections &Candida albicans
Subcutaneous fungal infections
Actinomycetes and Nocardia

CENTRAL NERVOUS SYSTEM INFECTIONS (CNS) – MI 5.1 TO 5.3

Meningitis:
Streptococcus pneumoniae
Streptococcus agalactiae
Neisseria meningitidis
Haemophilusinfluenzae Listeria
TB meningitis, and Spirochetal (T pallidum and Leptospira)
Cryptococcus
Poliovirus
Enteroviruses, Coxsackie virus, echovirus, Mumps virus and other viruses

Encephalitis:
Herpes viruses- HSV 1 and 2
Encephalitis group of Arboviruses: JE virus, West Nile fever Rabies
Nipah virus Slow
viruses
Toxoplasmosis
Primary amoebic Meningoencephalitis (Naegleria)
Granulomatous ameobic encephalitis (Acanthamoeba andBalamuthia)
Neurocysticercosis
85

GENITO-URINARY SYSTEM & SEXUALLY TRANSMITTED INFECTIONS – MI 7.1 TO 7.3

STIs:
Gonorrhoea, Non GonococcalUrethritis(NGU)
Trichomonasvaginalis
Bacterial vaginosis T
pallidum
H ducreyi
Lymphogranulomavenereum – C trachomatis
Granuloma inguinale
Viral- HBV, HCV,HIV, HSV 1& 2, HPV

Urinary tract infection:


E coli, K pneumoniae, Proteus spp, Enterococcus spp,Staphylococcus saprophyticus,
S agalactiae.

6. MODEL QUESTION PAPERS


86

PAPER I

(GENERAL MICROBIOLOGY, IMMUNOLOGY, CVS & BLOOD, RESPIRATORY TRACT


INFECTIONS, ZOONOTIC & MISCELLANEOUS, AETCOM)

Time: 3 hours Total Marks: 100

A. Multiple Choice Questions (MCQs) (20 X 1 mark = 20)

B. Long Answer Questions (LAQs) (3 x 10 marks = 30)


1. Enumerate agents used for disinfection and antisepsis (2)
a. Describe the process to sterilize a pair of surgical scissors (2)
b. How do you achieve high level disinfection of a bronchoscope (3)
c. Briefly discuss skin antisepsis (3)

2. A 10 year old boy developed difficulty in breathing and collapsed immediately following
IV penicillin injection.
a. Name the type of hypersensitivity reaction in the above case? (1)
b. Describe the immunological mechanism of this condition. (7)
c. Classify hypersensitivity reactions. (2)

3. A 40 year old man presented to the OPD with a history of low grade fever with evening
rise of temperature and productive cough since 2 months. He had a history of loss of
appetite and weight loss. His CXR showed a nodular infiltrate in the apical right upper
lobe.
a. How will you proceed in this case, according to the RNTCP algorithm?(2)
b. Enumerate the conditions wherein sputum CBNAAT is mandated?(2)
c. Describe in brief the pathogenesis of Pulmonary Tuberculosis (3)
d. State the definition, genetic mechanism and method to prevent development of MDR
TB?(3)
87

C. Short Answer Questions (SAQs) (10 x 5 marks = 50)


1. Draw and label the structure of a bacterial cell wall and enumerate mechanisms of
bacterial virulence.
2. State the importance of primary smear examination and give five examples of infections
that can be diagnosed by smear examination.
3. What is the role of Major Histocompatibility Complex in health and disease?
4. Describe the structure and functions of IgM.
5. What are the types of Hospital Acquired Infection? Enumerate Personal Protective
Equipment and give its uses.
6. What is MRSA? How do you prevent the transmission of MRSA?
7. Mention the clinical types of pulmonary aspergillosis and its risk factors.
8. Write briefly the laboratory tests for the diagnosis of malaria.
9. Mention four arboviral infections prevalent in India. Describe the pathogenesis and
laboratory diagnosis of dengue.
10. Describe and discuss the role of autonomy and shared responsibility as a guiding principle
in patient care (AETCOM – Module no 2.2)
---X---
88

PAPER II

(GASTROINTESTINAL AND HEPATOBILIARY, SKIN AND SOFT TISSUE, CNS,GENITO-


URINARY & SEXUALLY TRANSMITTED INFECTIONS, AETCOM)

Time: 3 hours Total Marks: 100

A. Multiple Choice Questions (MCQs) (20 X 1 mark = 20)

B. Long Answer Questions (LAQs) (3 x 10 marks = 30)

1. A 34 year old male presented with blackish discolouration of left foot, edema, crepitus with
a foul smelling serous discharge following major road traffic accident.
a. What is your probable diagnosis? Mention two common etiological agents. (1)
b. Describe the pathogenesis and laboratory diagnosis of the above condition. (7)
c. Outline its management. (2)

2. A 25 year old pregnant lady is admitted with complaints of fever, malaise and jaundice
since 5 days. She gives a history of hospitalization 4 years ago and having received a blood
transfusion. Presently, her liver enzymes are elevated; serum bilirubin is 20 mg/dl and
HBsAg positive.

a. Which marker will you test for to rule out acute infection? (1)
b. Mention the serological markers of hepatitis B virus with their significance. (5)
c. What are the routes of transmission of this virus? (2)
d. How will you prevent infection of the neonate born to HBsAg positive mother? (2)

3. A 3 year old male child presented with abdominal pain and diarrhoea for the past three
days. He also gave history of vomiting and diarrhoea with expulsion of worms from mouth
and anus. On examination the patient had bloating, absence of bowel sounds, abdominal
tenderness and palpable mass.
a. Which parasite is most likely to cause the above condition? Enumerate three other
intestinal nematodes. (2)
b. Describe its pathogenesis and life cycle. (3+3)
c. What is cutaneous larva migrans? (2)

C. Short Answer Questions (SAQs) (10 x 5 marks = 50)


1. A 25 year old male presented with an ulcer on the genitalia. Enumerate microorganisms
causing a genital ulcer. Discuss the specific tests for syphilis.
2. Write briefly on the laboratory diagnosis of enteric fever.
3. A 30 year old male is suspected to have cryptococcal meningitis. How will you proceed in
the laboratory to confirm the diagnosis?
4. Name fungi infecting the skin. How is dermatomycosis diagnosed in the laboratory?
89

5. Compare and contrast eumycotic and actinomycoticmycetoma.


6. Briefly discuss pathogenesis and immunoprophylaxis of rabies.
7. Describe the pathogenesis, complications and prophylaxis of Varicella Zoster.
8. State the life cycle of Taeniasolium and pathogenesis of cysticercuscellulosae.
9. Write briefly on the pathogenesis and laboratory diagnosis of hydatid disease.
10. What is informed consent? (AETCOM – Module no 2.5)

---X---
90

7. PONDICHERRY UNIVERSITY MICROBIOLOGY PRACTICALS DISTRIBUTION OF 100


MARKS
(AS PER THE CERTIFIABLE COMPETENCIES IN THE CBME)

A. Primary smear* for diagnosis along with problem solving exercise – Urine/ Respiratory/
Pus/ Body Fluids - by Gram Stain = 10 marks

B. Primary smear for diagnosis along with problem solving exercise – sputum– smear
positive or negative -by ZN Stain = 10 marks

C. Examination of parasite found in stool along with problem solving exercise - Stool
examination = 10 marks

D. Interpretation of the fungus by KOH, growth on SDA and LPCB mount along with
problem solving exercise – Mycology = 10 marks

E. Interpretation of two serological test along with problem solving exercise – Serology =
10 marks

F. Interpretation of two Antimicrobial Susceptibility Report along with problem solving


exercise – AST = 10 marks

G. OSPE for Hand Washing = 10 marks

H. OSPE for Personal Protective Equipment = 10 marks

I. VIVA = 20 marks

*Recommended that the primary smears for gram- stain to be of GPC/ GNC/ GPB/ GNB of
commonly encountered bacteria

---X---
91

FORENSIC MEDICINE

1. SUBJECT: FORENSIC MEDICINE MINIMUM MARKS FOR PASSING

THEORY – 1 PAPER OF 100 MARKS 50


THEORY 1 X 100 = 100 50
PRACTICALS = 100 (VIVA=20 MARKS) 50
---------------------------------------------------------------------------------------------------------
TOTAL = 200 100
---------------------------------------------------------------------------------------------------------
PRACTICALS = 80
VIVA =20
---------------------------------------------------------------------------------------------------------
TOTAL =100 50
---------------------------------------------------------------------------------------------------------

2. ELIGIBILITY IN INTERNAL ASSESSMENT

THEORY = 50 20
PRACTICAL = 50 (VIVA = 10 IF INCLUDED) 20
---------------------------------------------------
TOTAL =100 50
----------------------------------------------------

3. UNIVERSITY THEORY DISTRIBUTION OF MARKS

MCQs 20 x 1 = 20 MARKS
LAQs 3 x 10 = 30 MARKS
SAQs 10 x 5 = 50 MARKS
92

4. DISTRIBUTION OF CONTENT:

PAPER I – SUBJECT TOPICS + AETCOM (1 SHORT NOTE)

SAMPLE-I Total MCQs LAQs SAQs


Forensic Pathology (Medicolegal 17 2 10 5
Autopsy, Exhumation, Thanatology,
Asphyxial Deaths)
Legal Procedure and Medical 19 4 15
Jurisprudence
Sexual Jurisprudence 21 6 10 5
Forensic Psychiatry 06 1 5
Traumatology 16 1 10 5
Toxicology 16 6 10
AETCOM (5) 05 5
TOTAL 100 20 30 50

5. UNIVERSITY PRACTICALS DISTRIBUTION OF MARKS

VIVA = 20 MARKS (Four Examiners x 5 = 20 Marks)

SUBJECT EXERCISES: 20 Stations x 4 Marks = 80 MARKS

Time for each station: 5 minutes.

(1) Issue injury certificate in prescribed format.


(2) Issue drunkenness certificate with reasons.
(3) Sex determination from bones with reasons.
(4) Age determination from X-rays with reasons.
(5) Age certification from physical and dental examination with reasons.
(6) Examination of accused (potency certification) in a case of alleged sexual assault.
(7) Examination of the victim in a case of alleged sexual assault.
(8) Estimation of age of a given foetus.
(9) Prepare Viscera packing and labelling for a case of suspected poisoning.
(10) Issue Medical sickness certificate.
(11) Issue Fitness from medical leave certificate.
(12) Interpretation of autopsy report with reasons.
(13) Issue Medical Certification of Cause of Death in prescribed format.
(14) Examination of Weapon.
(15) Examination of Wet-Specimen.
(16) Examination of Photograph.
(17) Examination of a Poison.
(18) Examination of a Poison.
(19) Examination of a given Appliances.
(20) Examination of a given Autopsy instruments.
6. MODEL QUESTION PAPER - PAPER I
93

Forensic Medicine
Time: 3-hours Max. Marks: 100
ANSWER ALL QUESTIONS

Each Section to be answered in separate Answer Book


Illustrate your answer with suitable diagrams

1) A 16-year old unmarried girl was found hanging in her home from a ceiling fan with the
help of a nylon dupatta, the ligature mark was found over anterior aspect of the neck,
running obliquely to merge with the hair line posteriorly.
a. Write about the types of hanging. 2
b. Write the different causes of death in hanging. 2
c. Briefly describe the post-mortem appearances in the above case with special mention
about the signs of antemortem hanging. 6
2) As per an eyewitness account, one shop owner was shot at by an unknown person from a
close range with a pistol on the forehead and made an escape in a motorcycle. Now answer
the following questions.
a. Classify firearm. 3
b. Describe the composition of a rifled cartridge, with a neat labelled diagram. 3
c. With the help of a labelled diagram, briefly describe the findings of the wound of
entrance in this case. 4
3) A11-year old boy was allegedly sodomised by his neighbour. The boy later informed his
parents and the matter was reported to the Police. The Police registered the case and the
victim was brought to the Forensic Medicinedepartment for examination.
a. What are the other unnatural sexual offences 1
b. Describe the examination of a victim of non-habitual passive agent 6
c. What are the punishment for penetrative sexual assault and aggravated penetrative
sexual assault? 3

4) Write short notes on: 10x5=50 Marks


a) Definition of rape as per Criminal Amendment Act 2013. 5
b) Enumerate the different types of drowning and add as note on Pathophysiology of
wet drowning (2+3)
c) Describe the Civil and Criminal Responsibilities of a Mentally ill person
(3+2)
d) Medical and Medicolegal Duties of a doctor in suspected case of poisoning (2+3)
e) Define Professional Misconduct and enumerate with five examples (1+4)
f) Define Subpoena and describe the procedure of serving a summon (1+4)
g) Define medical negligence and briefly describe the defences against medical
negligence (1+4)
h) Enumerate the different types of Intracranial haemorrhages and describe the
intracranial haemorrhages that occurs outside the duramater (2+3)
i) Classify organophosphorous compounds and describe the clinical manifestations of
organophosphates (1+4)
j) What is informed consent and informed refusal in relation to a surgical operation
(3+2)
-oo0oo-
94

COMMUNITY MEDICINE

1. SUBJECT: COMMUNITY MEDICINE MINIMUM MARKS FOR PASSING

THEORY – 2 PAPERS 100 MARKS EACH40 IN EACH PAPER

THEORY 2 X 100 = 200 100


PRACTICALS = 100 (VIVA=20 MARKS) 50
---------------------------------------------------------------------------------------------------------
TOTAL = 300 150
---------------------------------------------------------------------------------------------------------
PRACTICALS = 80
VIVA = 20
---------------------------------------------------------------------------------------------------------
TOTAL =100 50
---------------------------------------------------------------------------------------------------------

2. ELIGIBILITY IN INTERNAL ASSESSMENT

THEORY = 50 20
PRACTICAL = 50 (VIVA = 10 IF INCLUDED) 20
---------------------------------------------------
TOTAL =100 50
----------------------------------------------------

3. UNIVERSITY THEORY DISTRIBUTION OF MARKS

MCQs 20 x 1 = 20 MARKS
LAQs 3 x 10 = 30 MARKS
SAQs 10 x 5 = 50 MARKS
95

4. DISTRIBUTION OF CONTENT:

The topics mentioned below are based on the MCI document by the Medical Council of India
titled “Competency based undergraduate curriculum for Indian Medical Graduate, 2018-
Volume II”. The numbers mentioned in the brackets is the weightage of each of the topics out
of the total of 100 marks. This weightage distribution was developed by taking an average of
the weightage given by a panel of 4 subject experts independently.

PAPER I –Demography and vital statistics, Reproductive, maternal and child health, Nutrition,
Concept of health and disease including history of Medicine, Relationship of social and
behavioural to health and disease, General epidemiology include screening, Basic statistics
and its applications, Environmental health problems include Biomedical waste, Disaster
Management +AETCOM (1 SHORT NOTE)

No. Topics as per the MCI competency list Marks MCQ LA SAQ
Q
Demography and vital statistics (10) 2
1 Reproductive, maternal and child health (20) 40 4 1 4
Nutrition (10) 2
Concept of health and disease including
history of Medicine (15) 3
2 20 1 1
Relationship of social and behavioural to 1
health and disease (5)
General epidemiology include screening (15) 4
3 20 1 1
Basic statistics & its applications (5) 1
Environmental health problems include
2
4 Biomedical waste 10) 15 0 3
1
Disaster Management (5)
5 AETCOM (5) 5 - - 1
6 Total 100 20 30 50

PAPER II – Epidemiology of communicable & non-communicable diseases, Health planning


and management, Health care of the community, Principles of health promotion and education,
Occupational Health, Mental health, Geriatric services, International health, Essential
medicine, Recent advances in community medicineAETCOM (1 SHORT NOTE)

No. Topics as per the MCI competency list Marks MCQ LAQ SAQ

Epidemiology of communicable & non-


1 30 6 1 3
communicable diseases (30)
Health planning and management (10)
2 Health care of the community (10) 30 6 1 2
Principles of health promotion &education (10)
3 Occupational Health (10) 10 2 1 -
4 Mental health(5) 10 2 - 2
96

Geriatric services (5)


5 International health (5) 5 1 - 1
Essential medicine (5)
6 10 3 - 1
Recent advances in Comm. Med.(5)
7 AETCOM (5) 5 - - 1
Total 100 20 30 50

Choose Set 1 or set 2 to design the question paper.

5. UNIVERSITY PRACTICALS DISTRIBUTION OF MARKS

VIVA = 20 MARKS
SUBJECT EXERCISES TOTAL = 80 MARKS
Case discussion - 40 marks
Statistics and EpidemiologicalExercises(2): 20 marks
5 Spotters: 10 marks
2 observed OSCE/ OSPE: 10 marks

Statistics: measures of central tendency, measures of dispersion, vital statistics, indicators of


morbidity and mortality

Epidemiological exercises: measurement of risk in various study designs, screening tests,


investigation of epidemic, assessment of vaccine requirement, indicators of specific diseases like
malaria, filarial, tuberculosis, calculation of chlorine demand

Spotters: nutrition, vaccines, entomology, pesticides, disinfectants, drugs used for common
illnesses like TB, diabetes, hypertension, diarrhoeal diseases, and conditions like anaemia,
contraceptives, records and cards used in National Health programmes.
Questions for the spotters should be designed to assess understanding and application of the
knowledge in a particular topic and not just recall.

OSCE/OSPE: communication skills to elicit specific history, Counselling skills for eg.
Contraceptives, complementary nutrition, before performing lab tests, clinical skills of
measuring blood pressure, measurement of height & weight and calculation & interpretation
of BMI in adults, nutritional assessment in children using anthropometry and growth charts,
assessment of pallor and interpretation, assessment of dehydration, examination of diabetic
foot, administration of a vaccine through a specific route of administration in a simulated
environment (model/ mannequin if available)

6. MODEL QUESTION PAPER - PAPER I& PAPER II


97

Pondicherry University
Community Medicine Paper 1
Time: 3 hours Total marks: 100
Note : All the questions are compulsory.
Draw suitable diagrams wherever necessary.

I. MCQs (1 X 20 = 20 marks)
II. Long answer questions (10 x 3 = 30 marks)
1. A 25 year ole woman approaches the PHC Medical officer regarding advice on
contraceptives. Her only child is 4 weeks old and she wants to have her next child after
a year.
a. Define Family planning. Enumerate the various contraceptives. (2 + 3)
b. Which method would you suggest for her and why? (3)
c. What is unmet need for contraception? (2)

2. Define natural history of disease. Discuss the various levels of prevention and the modes
of intervention using Diabetes as an example. Describe the iceberg phenomenon.
(2+5+3=10 marks )

3. A study was conducted to find the association between tobacco use and oral cancer in
which 100 cases of oral cancer were compared with 100 persons without oral cancer.
a. What type of study is this? (1)
b. How will you interpret the strength of association in this study? (3)
c. What are the various types of bias that can be expected in this type of study and
how can you minimise them? (6)

III. Short answer questions: (5 x 10 = 50 marks)


a) Color coding of Biomedical waste Management as per 2016 Guidelines
b) Sanitation barrier and its significance
c) Importance of triage in disaster management
d) Types of epidemic
e) Importance of incubation period
f) Demographic cycle
g) Anemiamukht Bharat
h) High risk pregnancy
i) Balanced diet
j) Role of physician in health care

----------X----------
98

Pondicherry University
Community Medicine Paper 1
Time: 3 hours Total marks: 100

Note : All the questions are compulsory.


Draw suitable diagrams wherever necessary.

I. MCQs (1 X 20 = 20 marks)
II. Long Answer Questions(LAQs) (10 x 3 = 30marks)

1. A 26 year old woman, belonging to a nuclear family living with her husband and two
children aged 4years and 2years, came to chest clinic with C/c of cough for more than
2weeks loss of appetite and lost of body weight and not responding to conventional
antibiotics. There was no past h/o tuberculosis. However, the treating physician
suspected Pulmonary TB and she was sent for sputum microscopy.

(a.) Describe the latest RNTCP guidelines for classification, diagnosis and
treatment for this patient. (2+3+2=8marks)
(b.) Describe the points that are to be covered during the health education sessions
to the patient. (2marks)

2. a.) Define “Objective”, “Target” and “Goal”. (3+4+3 =10marks)


b.) Describe in detail the different steps in the planning cycle.
c.) Enumerate the basic steps that are involved in the evaluation of Health
Services.
3. What are the various types of Occupational diseases? Describe briefly the
occupational cancers. (7+3 = 10marks)

Short Answer Questions(SAQs) (5x10=50 marks)

a. Modified Jones Criteria for the diagnosis of Rheumatic Heart Disease.


b. Specific responsibilities of World Health Organization (WHO) for establishing and
promoting international standards in the field of Health.
c. Primary health care and its principles
d. Ayushman Bharat
e. Post-exposure prophylaxis in Rabies
f. Criteria for clinical diagnosis of Dengue Hemorrhagic Fever (DHF).
g. Barriers to effective Communication (during Health Education).
h. Classify the various health problems seen among the geriatric population.
i. Objectives and Strategies of National Mental Health Program .
j. What are the instances in which the confidentiality of patient information may be
breached?

--------
99

OTORHINOLARYNGOLOGY - MBBS THEORY,CONTENT & MARK DISTRIBUTION

SYSTEM WISE MCQ LAQ SAQ MARKS


DIVISION SYSTEM-WISE

Otology 7 1 3 32
Rhinology 6 1 3 31
Oral Cavity, 7 1 3 32
Pharynx,Larynx,
Head & Neck
AETCOM - - 1 5

TOTAL MARKS 20 30 50 100

80% of the questions to be from core topics


MCQ’s- Clinical Scenarios- 4 options with single best response
Timings (3 hours )
20 MCQ -30 minutes
3 LAQ - 2.30 hrs for LAQ &SAQ
10 SAQ
100

OTORHINOLARYNGOLOGY MBBS MODEL THEORY QUESTION PAPER

ANSWER ALL THE QUESTIONS

ILLUSTRATE YOUR ANSWERS WITH SUITABLE DIAGRAMS

Time: 2 ½ hours Maximum Marks : 80

1. A 35 year old female presents with intermittent mucopurulent, non-foul smelling and profuse
discharge from the left ear for 2 years, aggravated by common cold, associated with decreased
hearing (1+3+3+3=10 marks)
a) Identify the clinical condition.
b) Describe the clinical examination findings.
c) How will you evaluate this patient?
d) Discuss the medical and surgical management of this patient.

2. A patient with carcinoma larynx presents to emergency department with respiratory distress
and noisy breathing (1+4+5=10 marks)
a) What is the immediate surgical procedure to relieve his symptoms?
b) Describe the steps of the above surgical procedure.
c) Discuss the differential diagnosis of stridor in adults.

3. Thirty year old gentleman with persistentnasal obstruction is diagnosed to have deviated nasal
septum. (4+3+3=10 marks)
a) What are the surgical options for this condition with their indications & differences?
b) Discuss the complications of the septal surgeries.
c) Describe the etiopathology & complications of septal deviation.

4. Write Short notes on (10x5=50 Marks)


A. Illustrate the medial wall of middle ear? What are its clinical implications?
B. Discuss the pathology and management of Benign Paroxysmal Positional Vertigo.
C. How will you manage a 2 year old child with congenital hearing loss?
D. Outline the management of persistent epistaxis in a 50 year old gentleman.
E. Tabulate the difference between antrochoanal polyp and Ethmoidal polyps in terms of
etiology, clinical features and management.
F. How will you investigate and treat a patient with CSF rhinorrhea?
G. What is the etiopathology and management of Zenker's diverticulum?
H. How will you evaluate a patient present with white patch on tonsils?
I. Explain the pathways of spread and management of Nasopharyngeal angiofibroma.
J. How will you counsel the parents of a 6 year old child posted for denotonsillectomy?
101

OTORHINOLARYNGOLOGY MBBS PRACTICAL MARK DISTRIBUTION


PRACTICAL
SL.NO MARKS TIMING
1 1 Long Case 1x35= 35 marks 20 minutes
2 1 Short case 1x25= 25 marks 10 minutes
Total – 30 minutes
3 4 OSCE (Spotter and Skill 4 x5= 20 marks 5 Minutes each
assessment)

Total 20 minutes
TOTAL 80 Marks 50 Minutes

ORALS

ORAL MARKS

Instruments 5
X-rays 5
Osteology/Specimen 5
VIVA 5
TOTAL 20 marks
102

OPHTHALMOLOGY
THEORY EXAM 100 MARKS

MCQ: 1 x 20 = 20MARKS (10 Anterior segment+ 10 Posterior segment)


LAQ: 3 x 10= 30 MARKS
SAQ: 10 x 5 = 50 MARKS

LAQ –3 *10= 30 marks


1) CORNEA:

Corneal ulcer – Bacterial, Fungal, Viral, Keratoplasty

2) LENS:

Congenital and acquired cataract, its management and complications

3) GLAUCOMA:

Primary open angle glaucoma

Primary angle closure glaucoma

Congenital glaucoma

4) Uvea:

Anterior Uveitis

5) Lacrimal Apparatus:

Congenital and Acquired Dacryocystitis

6) Acute Red eye:

Conjunctiva/Cornea/Glaucoma/Iridocyclitis

7) RETINA:

Diabetic Retinopathy, Retinal detachment, Retinoblastoma, Retinitis Pigmentosa,


Central Retinal Artery Occlusion, Central Retinal Vein Occlusion

8) NEURO OPHTHALMOLOGY:

Visual pathway and its lesions, Optic neuritis, Papilledema, Optic atrophy
103

9) OCULAR TRAUMA:

Blunt trauma

10) REFRACTION AND OTHER ANOMALIES:

Myopia

11) COMMUNITY OPHTHALMOLOGY:

National Programme for Control of Blindness, Eye Banking

SAQ: 10 X 5 = 50 marks

1) EMBRYOLOGY, ANATOMY AND PHYSIOLOGY:

Development of lens and retina, Wald’s visual cycle, Aqueous Humour secretion &
Drainage, Accommodation
2) LIDS:

Ptosis, Entropion, Ectropion, Lagophthalmos, Symblepharon, Trichiasis, Blepharitis,


HordeolumInternum, HordeolumExternum, Chalazion

3) ORBIT:

Proptosis, Pre-septal and Orbital cellulitits,

4) LACRIMAL APPARATUS:

Dacryocystitis (Acute, Chronic, Congenital)

5) CONJUNCTIVA:

Infective, Allergic, Cicatricial, Pingecula, Pterygium, Symblepharon, Sub-


conjunctivalH’age, Bitot’s Spots

6) SCLERA:

Episcleritis, scleritis, staphyloma

7) CORNEA:

Corneal Ulcer – Clinical features/ Complications/Management, Corneal opacity,


Keratoconus, Interstitial keratitis, Pannus
104

8) LENS:
Complicated Cataract, Traumatic Cataract, After Cataract, Management of Congenital
Cataract, Endophthalmitis, Panophthalmitis, Subluxation and Dislocation of lens,
Intra-Ocular Lenses, Lens induced Glaucoma
9) UVEA:

Anterior uveitis – Keratic Precipitates/ Iris nodules/Complications of Anterior Uveitis/


Management of Anterior Uveitis/ Hypopyon, Sympathetic Ophthalmitis, PhthysisBulbi

10) GLAUCOMA:
Lens induced glaucoma, Malignant glaucoma, Anti- glaucoma medications, Field defects
in Glaucoma, Clinical features of PACG (Primary angle closure Glaucoma),
Buphthalmos
11) VITREOUS:
Vitreous haemorrhage, AstroidHyalosis, SynchisisScintillans
12) RETINA:

Diabetic Retinopathy – Clinical Features of DR/ Treatment of DR, Retinal Artery


Occlusion, Retinal Vein Occlusion, Hypertensive Retinopathy, Retinopathy of
prematurity, Retinitis Pigmentosa, Central Serous Retinopathy,Cystoid Macular
Edema , Age Related Macular Degeneration, HIV Retinopathy

13) NEURO OPHTHALMOLOGY:

Marcus Gunn Pupil, Toxic/ Nutritional Optic Neuropathy.

14) OCULAR INJURIES:

Berlins Edema, Chemical injuries, Sympathetic Ophthalmitis

15) REFRACTION AND OTHER ANOMALIES:

Hypermetropia, Astigmatism, Presbyopia, Aphakia, Contact Lens, Surgical


Management of Myopia

16) COMMUNITY OPHTHALMOLOGY:

Childhood Blindness, Corneal Blindness, Legal Blindness, Vision 2020, Vitamin A


Deficiency, School Eye Screening, District Blindness Control Society, Rehabilitation of
the Blind

17) MISCELLANEOUS:
Ocular manifestations of systemic disorders – Thyroid/DM/TB/Leprosy/Syphilis/HIV,
Lasers in Ophthalmology, Ocular Pharmacology
18) AETCOM:
105

OPHTHALMOLOGY - MODEL QESTION PAPER


THEORY -100 MARKS
MCQ’s: 1x20 = 20 MARKS

LAQ: 3 x 10 = 30 MARKS
SAQ: 10 x 5 = 50 MARKS

I) LAQ – 1 *10 = 30 MARKS :

I) A 60 year old male patient presents to the OPD with complaints of painless progressive
dimision of vision for past two years.

a) Ennumerate the causes for the same (1 mark)

b) Senile cataract - Discuss the etiology, symptoms, signs, stages and investigations
(4marks)

c) What are the different types of cataract surgery? Write the steps of Small incision /
Phacoemulsification cataract surgery (4 marks)

d) List the post operative complications (1 mark)

II) A 55 year old male patient who is a farmer by occupation, presented to the OPD with
complaints of sudden onset of pain, redness and defective vision in right eye for 2 day
duration. He gives a H/O injury with vegetative matter 2 days prior to the onset of
complaints. He is a known diabetic of 20 years duration on irregular treatment.

a) What is the most probable diagnosis? 2 marks

b) Describe the classification and clinical features? 4 marks

c) Describe the treatment modalities? 4 marks

III) A mother brings her 3 year old child to the OPD with complaints of white reflex in both
eyes.

a) Enumerate the differential diagnosis of white reflex in both eyes. 2 marks


b) Discuss the heredity, clinical features & management of retinoblastoma.
( 1+4+3 = 8 marks )
106

III) SAQ – 10 x 5 MARKS =50 MARKS

1) A 50 year old male patientcame with complaints of painless defective vision in the right
eye, for 5 years. On examination, visual acuity in (RE)is PL+ and PR accurate.There is a
total leucomatous corneal opacity in (RE).Ultrasound of (RE) shows attached retina.

a) What are the causes for corneal opacity?


b) What are the treatment options to restore vision in this patient?
c) What are the indications and contra indications of eye donation?
d) What are the methods of corneal preservation?

2) A 62 year old female patient presented to the casualty with complaints of sudden onset
of severepain, redness and defective vision in the Right eye. She also gave a history of
developing the above complaints, while she was watching movie in a dimly illuminated
room.

a)What is your most probable diagnosis ?


b) What are the signs and symptoms?
c) How will you differentiate the above condition from acute iridocyclitis?
d) What is the immediate treatment?
e) What is the definitive treatment?

3) 20 days old newborn is bought to the OPD with complaints of redness, purulent
discharge, and lid swelling of two days duration.
a)What is the differential diagnosis of the above condition?
b) What are the causative organisms and incubation period?
c) What is the treatment?

4) A 30 year old female came to the OPD with complaints of forward protrusion of eyeballs
of 1 month duration .It was painless in nature and not associated with defective vision.
a)What is the differential diagnosis?
b) How will you investigate this patient?
5) Describe the development of lens.
6) 10 year old boy came with history of injury with cricket ball and came pain redress and
defective vision
a) What are the ocular manifestations of blunt trauma?
7) What is Vision 2020
a) What are the goals and objectives?
b) What is the expected blindness rate by 2020 and measures to achieve this rate?
107

8) A 68 year old female patient complains of inability to see objects coming from either
side.On examination of the visual field, patient has right homonymous hemianopia with
sparing of macular vision
a) Where is the site of lesion?
b) Draw the visual pathway and describe the lesions at various levels.
9) A 7 year old child complains of inability to see the blackboard but has no problem in
reading her books.
a) What is the refractive error she has?
b) What are the clinical types and various treatment modalities for this condition?
10) What are the instances in which confidentiality of patient information may be breached?

OPHTHALMOLOGY –PRACTICALS - 100 MARKS


Long cases 1: 2 x 30 = 60 marks
OSCE 10:10 x 2 = 20 marks Viva: =
20 marks
Total: 100 marks
108

GENERAL MEDICINE

1. SUBJECT: GENERAL MEDICINE MINIMUM MARKS FOR PASSING

THEORY – 2 PAPERS 100 MARKS EACH 40 IN EACH PAPER


THEORY 2 X 100 = 200 100
PRACTICALS = 200 (VIVA=40 MARKS) 100
---------------------------------------------------------------------------------------------------------
TOTAL = 400 200
---------------------------------------------------------------------------------------------------------
PRACTICALS = 160
VIVA = 40
--------------------------------------------------------------------------------------------------------
TOTAL = 200 100
---------------------------------------------------------------------------------------------------------

2. ELIGIBILITY IN INTERNAL ASSESSMENT

THEORY = 50 20
PRACTICAL = 50 (VIVA = 10 IF INCLUDED) 20
---------------------------------------------------
TOTAL =100 50
----------------------------------------------------

3. UNIVERSITY THEORY DISTRIBUTION OF MARKS

MCQs 20 x 1 = 20 MARKS
LAQs 3 x 10 = 30 MARKS
SAQs 10 x 5 = 50 MARKS
109

4. DISTRIBUTION OF CONTENT:

PAPER I – SYSTEMIC MEDICINE +AETCOM

PAPER I MCQs LAQs SAQs


CARDIOLOGY 2 1 1
NEUROLOGY 4 1 1
GASTROENTEROLOGY 4 2
& LIVER DISEASE
RESPIRATORY SYSTEM 4 1 1
NEPHROLOGY & FLUID 4 2
AND ELECTROLYTES
CARDINAL SYMPTOMS 2 2
AETCOM 1
TOTAL 20 30 50

PAPER II – TROPICAL MEDICINE, INFECTIOUS DISEASE INCLUDING


TUBERCULOSIS, DERMATOLOGY AND LEPROSY, PSYCHIATRY AND CRITICAL CARE
MEDICINE + AETCOM

PAPER II MCQs LAQs SAQs


TROPICAL & 3 1
ENVIRONMENTAL MEDICINE
INFECTIOUS DISEASE, 1 1
TUBERCULOSIS
ENDOCRINOLOGY & 1 1
DIABETES
HEMATOLOGY 1 1
GERIATRICS/GENETICS 2 1
NUTRITION 2 1
IMMUNOLOGY & 3 1
MUSCULOSKELETAL
CRITICAL CARE MEDICINE 2 1
DERMATOLOGY,STD,LEPROSY 3 2
PSYCHIATRY 2 2
AETCOM 1
TOTAL 20 30 50
110

MODEL QUESTION PAPER


GENERAL MEDICINE – PAPER I (SYSTEMIC MEDICINE)
TOTAL MARKS: 100 MARKS TIME : 3 HOURS (30 MINS FOR
MCQS)

LONG ANSWER QUESTIONS 3 X 10 = 30MARKS

1. Describe the etiopathogenesis, clinical features,investigations and treatment of infective


endocarditis. [ 3+3+2+2= 10]
2. 47 year old car driver brought to the casualty with history of generalised seizures since past
one hour with no such past history. On examination he was drowsy, with laboured breathing
and having continuous seizures. [1+3+4+2=10]
I. What is the diagnosis?
II. List out common precipitating factors for it.
III. Describe how you will manage this patient.
IV. What are the complications of this condition?

3. Elaborate on pathophysiology, clinical features, investigations and management of COPD.


[ 2+3+2+3= 10]

SHORT ANSWER QUESTIONS 10 X 5 = 50 MARKS

1. Write about the causes/risk factors, clinical features and treatment of pyelonephritis.
2. Enumerate the role of fibrin-specific thrombolytic therapy in ST-elevatedacute
myocardial infarction,contraindications and for complications of thrombolytic therapy.
3. What are the causes of cirrhosis? Write a note on the etiopathogenesis of NASH.
4. Write a step wise approach (clinical/biochemical parameters) in evaluation of
hypokalemia.
5. Explain the etiopathogenesis, clinical features and treatment of pneumonia.
6. Thirty five year old male brought with leg swelling and breathlessness to hospital
diagnosed as CKD.
a) How will you counsel about the financial expenses ?
b) Write about medico legal ethics in human organ transplantation
7. Define syncope and write the causes and evaluation of syncope.
8. Describe the clinical manifestations, immuno-pathogenesis and
treatmentofGuillainBarré Syndrome.
9. How to approach a patient with cyanosis?
10. Compare and contrast clinical features and management of Ulcerative colitis and
Crohn’s disease.
111

GENERAL MEDICINE – PAPER II


(TROPICAL MEDICINE, INFECTIOUS DISEASE INCLUDING TUBERCULOSIS,
DERMATOLOGY AND LEPROSY, PSYCHIATRY AND CRITICAL CARE MEDICINE)
MARKS : 50 SYSTEMIC MEDICINE +50 MARKS ALLIED SUBJECTS TIME: 3 HOURS( 30
MINS FOR MCQS)
LONG ANSWER TYPE QUESTIONS 3 X 10 = 30 MARKS

1. A 20-year-old male was admitted with fever and headache for 10 days. He had history of
recurrent episodes of vomiting. On clinical examination, he was disoriented, had diplopia
and neck stiffness. [1+3+4+2=10]
a. What is the most probable diagnosis?
b. Discuss the investigations with their interpretation that will help clinch the
diagnosis.
c. What is the treatment of this disease?
d. List four complications of this disease.

2. A 65-year-old man was brought to casualty in an unconscious state. He is a diabetic for 10


years and stopped his medications a week back. He also had vomiting & abdominal pain
for 2 days. On examination, he was drowsy and dehydrated, with a pulse rate of 110/min
and BP of 90/60 mm of Hg. His blood glucose was 685 mg/dl. [2+3+3+2=10]

a. What are the two diagnostic possibilities?


b. Describe the pathogenesis of any one of the diagnosis.
c. Discuss the clinical features of the above diagnosis mentioned in (b).
d. How will you manage the condition you described in (b)?

3. A 22 yr old female presented with chronic diarrhea and pins and prick sensation in both
her lower limbs.On examination she had mild icterus and severe pallor. [ 2+3+3+2= 10]

a. What is the most probable diagnosis and its causes?


b. How to evaluate the patient?
c. What will be the peripheral smear picture in this patient?
d. How to treat this patient?

SHORT ANSWER TYPE QUESTIONS 10X 5 = 50MARKS

1. Geriatric rehabilitation.
2. What are the diseases which can mimic schizophrenia? Explain some of the antipsychotics
used in schizophrenia with their side effects.
3. How will you confirm and manage a patient who presented with a hypopigmented ,
anaesthetic patch on his forearm?
4. How to approach a patient presenting with genital ulcer?
112

5. How will you evaluate a patient with hypotension and explain why noradrenaline is the
preferred vasopressor in septic shock.
6. How should a doctor deal with the emotions of patients and family facing death? Can doctors
assist death?
7. How do you evaluate a patient with significant unintentional weight loss?
8. Explain the pathogenesis of Rheumatoid arthritis. Enumerate the extra-articular
manifestation of rheumatoid arthritis.
9. Define fever and hyperthermia.Enumerate the causes and treatment of hyperthermia.
10. What are the psychiatric aspects of alcohol use disorder?

5. UNIVERSITY PRACTICALS DISTRIBUTION OF MARKS

VIVA = 40 MARKS

SUBJECT EXERCISES = 160 MARKS


1. LONG CASE= 60 MARKS
2. SHORT CASE= 2 X 30 = 60
3. OSCE= 20 ( SKILLED ) +20 (UNSKILLED) = 40 MARKS

6. MODEL QUESTION PAPER - PAPER I& PAPER II

----------X----------
113

SURGERY

Paper I
MCQ 20 Marks
LAQ 10 X 3 = 30Marks
SAQ 5 X 10 = 50 Marks
General surgery
Surgical Principles
Wound healing
Special infections
Trauma, blood transfusion
Burns
General oncology
Breast & Endocrine
Surgical Care (Minor Procedures)
Paediatric surgery
Plastic surgery

Paper II
MCQ 20 Marks
LAQ 3 X 10 = 30 Marks
SAQ 10 X 5 = 50 Marks

GI surgery
Urology
Neurosurgery
CTVS
Radiotherapy
Physical medicine
Radiology
Orthopaedics
Anesthesia
Dentistry
114

Model Paper 1

MCQ 20 Marks

Q. Questions Marks Subject


No. section
1 1. A 70-year-old man has been sent to the emergency Gen Surg
department from a nursing home, complaining of
intermittent sharp abdominal pain. He has not opened his
bowels for 5 days. He suffered a major stroke in the past
and requires constant nursing care. He has a history of
chronic constipation. Previous medical history includes
chronic obstructive airways disease for which he is on
regular inhalers. He is allergic to penicillin and is an ex-
smoker.
His blood pressure is 110/74mmHg and the pulse rate is
112/min. His temperature is 37.8°C. There is gross
abdominal distension with tenderness, most marked on the
left-hand side. The abdomen is resonant to percussion and
digital rectal examination reveals an empty rectum.
a) What would the abdominal X-ray show 1
b) What other radiological investigation could be 2
employed if the diagnosis was in doubt
c) How should the patient be managed 4
d) What is the explanation for the pathology 3
2 2. A 40-year-old woman has been referred to the surgical Endocrine
outpatients with a painless lump in the neck. She had
noticed the lump 2 weeks previously when looking in the
mirror. She had not noticed any other lumps and does not
complain of any other symptoms. She has not gained or
lost any weight recently and her bowel habit has remained
normal. Examination reveals a solitary 2 x 2cm swelling to
the left of the midline just above the manubrium. The
swelling is firm, smooth and fixed. The swelling moves on
swallowing, but does not move on protrusion of the tongue.
There are no associated palpable lymph glands. General
examination reveals no further abnormalities.
a) What is the differential diagnosis for a lump in the
anterior triangle of the neck 2
b) Where is this lump likely to be originating from
c) What steps would you take in the assessment of this lump 1
d) Which factors may suggest malignancy 3
e) What are the commonest types of malignancy
2
2
115

3 1. 60 year old male presents with palpable lump in the Gen


right hypochondrium associated with jaundice Surg/GI
a. What is the probable diagnosis and other 3
significant clinical features
b. Discuss the evaluation of this patient 4
c. Discuss in brief the treatment options 3

4.a. 30 year bank employee met with road traffic accident is being 5 Blood
resuscitated in emergency department received 8 units of trans
blood transfusion within 2 hours, discuss the complications

b. 65 year old male presents with dribbling of urine with 5 Uro


suprapubic fullness, describe the management

c. 45 year old female presents with 2cm by 3 cm lump in right 5 Breast


breast, discuss the evaluation

d. 5 year old child presents to ED with history of swallowing two 5 Paed


battery cells half an hour back with no other complaints,
discuss the management

e. 26 year male comes with a clean ulcer over the left leg of 5 x 8 5 Plast /Gen,
cm, what is the appropriate management
f. Young female of 23 years presents to OPD with a3cm by 3cm 5 Endo
nodule in the right lobe of thyroid with a prescription of L-
thyroxine 50 micrograms from a practitioner, discuss the
evaluation and management

g. 8 week child with projectile vomiting for last 4 days 5 Paed Surg
dehydrated and emaciated as brought to surgery OPD, with a
visible peristalsis what is the diagnosis, discuss the treatment
in brief

h. Explain to the patient who is undergoing below knee 5 AETCOM


amputation
i. Explain to the patient who is undergoing abdominoperineal 5 Do
resection for Ca Rectum management of colostomy

j. Describe the management of a tense swollen right leg 5 Gen


following a snake bite
116

Model Paper 2

MCQ 20 Marks

1 A 38-year-old computer engineer is referred to surgical outpatients Genito


complaining of pain in the right groin. He has noticed this over the urinary
last few months and his pain is worse on exertion. He has also
noticed an intermittent swelling He is a smoker of 25 cigarettes per
day and drinks 10 units of alcohol per week. He is apyrexial with
normal blood pressure and pulse. The abdomen is grossly normal but
there is some tenderness in the right groin. The patient is asked to
stand. In the right groin, there is a swelling which is more
pronounced when the patient coughs. The other groin and the scrotal
examination are normal.
a) What is the likely diagnosis 2
b) What are the anatomical boundaries 2
c) What are the complications associated with this condition 3
d) How should the patient be treated 3
2 A 22-year-old woman presents to the emergency department GI
complaining of lower abdominal pain. This has steadily increased in
severity over the previous 24h and woke her from her sleep. The
pain is constant, and simple analgesia has not helped. She has
vomited once in the department. Her menses are regular and she is
now on day 12 of her cycle. There is no history of vaginal discharge
or urinary symptoms. She has no children. She has not undergone
any previous surgery. There is no other relevant medical history.
She takes no current medication and has no allergies. She is a non-
smoker. On examination Her blood pressure is 110/72mmHg and
pulse rate is 110/min. Her temperature is 38.2°C and there is lower
abdominal tenderness, more marked in the right iliac fossa, with
some rebound tenderness. There are no palpable masses and the
loins are not tender. Digital rectal examination is normal. Bimanual
per vaginal examination reveals adnexal tenderness on the right.
a) What is the differential diagnosis 2
b) How should the patient be managed initially 3
c) If you are unsure of the diagnosis, how should you proceed 5
3 46- year-old shop keeper presents to OPD with pain and bogginess of Vascular
the limbs over the day
a) What is the most likely clinical conditi on 2
b) What are the relevant investigations 4
c) Outline the treatment plan 4
4.a 56 Year old male presents with history of blood and mucus in the 5 GI
stools for the last two months, briefly discuss the evaluation
b 42 year old dye worker presents with painless hematuria of 3 weeks 5 Urology
duration,describe the possible differential diagnosis and evaluation
c Following laparoscopic appendicectomy a college student presents 5 Plast /Gen
with an ugly scar over the umbilical port site, what is the diagnosis
and how will you manage
117

d 36 year old steno presents with a 3 cm sized swelling over the left 5 Plast /Gen
wrist discuss the etiopathology and management
e An intern in an ICU inadvertently added KCL ampoule to a Ringer 5 Anesthesia
lactate drip which is about to be started, what is the consequence &CC
and what is the composition of the solution
f Explain the pathology and management of a patient with a dinner 5 Ortho
fork deformity after fall on outstretched hand
g Describe the method of scrubbing before an operative procedure 5 Gen
h A patient with a diagnosis of intestinal obstruction how will you 5 Gen/GI
pass the Ryle’s tube
i In the operation theatre the scissor slipped from the surgeon’s hand 5 Gen Princi
how will you sterilise it before using it again, describe the properties
of the agent /method used
j Explain to the relatives of a young man who is a polytrauma patient 5 AETCOM
being managed in a high dependency ICU following surgery whose
condition is critical

General Surgery Practical Examination

Long case : 1 x 50 = 50 Marks


Short case : 2 x 25 = 50 Marks
Oral (Viva-voice) : 4 x 10 = 40 Marks
OSCE : 4 X 15 = 60 Marks
--------------------------
Total : 200 Marks
---------------------------
118

OBG
Theory exam- total marks 200
Paper I- Obstetrics including social obstetrics and demography - maximum marks -100
Paper II- Gynaecology including family welfare - maximum marks -100
MCQ-20*1=20 marks

LAQ- 10*3=30 marks

SAQ- 10*5=50 marks

Practical examination Total marks -200

Subsections Marks allotted


OSPE/OSCE 60
CLINICAL EXAMINATION
Obs. Long Case 50
Gynaec.LongCase 50
ORAL EXAMINATION (Structured)
Obs 20
Gynae 20
119

FINAL YEAR MBBS EXAMINATION

MODEL QUESTION PAPER

PAPER – I

(OBSTETRICS INCLUDING SOCIAL OBSTETRICS)


Time: 3 hours Maximum: 100 Marks

(including MCQs)

Draw neat diagrams where necessary. All questions are compulsory

3X10 30 marks
I. A 32 years old Gravida 3 Abortion 2 is at38 weeks of gestation with GDM on Insulin
How will you manage this patient during labor ? 5 marks
How will you do periconceptional counselling in a women with type II DM ?
5marks
II. 25 years primigravida presents to the antenatal clinic at 36 weeks with over distended
uterus

a) List the causes of uterus being more than gestational age. 3 marks
b) Outline the management in dichorionic diamniotic twin pregnancy 7 marks

III. A 34 year old primiparous lady had operative vaginal delivery with perineal lacerations .
Her labour was induced afterprolonged prelabour rupture of membranes . Three days post
delivery she presented with history of fever of 39 degree centigrade for past 24 hours.

a) Write the diagnosis and causes for fever. 4marks


b) how will you manage this case? 6 marks

IV. Write short answers for the following: (5X10=50)


1. Write briefly on the cardiovascular changes in normal pregnancy (5)

2. A 25 year old second gravid is not compliant with oral iron and her hemoglobin is 9 gm%.
How will provide Iron therapy to this patient? (5)

3. A 35 year old primigravida comes to casualty with B P of 170/110. Investigations suggest


HELLP Syndrome. How will you diagnose and manage this patient? (2+3=5)
4. Define and enumerate the causes of maternal mortality (2+3=5)
120

5. A primigravida reports to the ante natal clinic at 8 weeks of gestation. What schedule of
antenatal care would you advise her? What are the advantages of ante natal care?
(2+3=5)
6. A patient in prolonged labour has excessive bleeding half an hour after delivery. What is
the diagnosis? What measures are taken to prevent atonic postpartum hemorrhage?
(1+4=5)
7. What are the indications and pre requisites for Prophylactic outlet forceps
(3+2=5)
8. A primigravida presents with lower abdominal pain a with positive urine pregnancy test.
Ultrasound reveals tubal ectopic pregnancy. Patient does not want surgical management.
What is the non surgical method of managing this patient? (5)

9. What are the principles of newborn resuscitation? (5)

10. Prostaglandins for induction of labor. (5 )


121

FINAL YEAR MBBS EXAMINATION


MODEL QUESTION PAPER
PAPER – II

(GYNAECOLOGY INCLUDING FAMILY WELFARE)


Time: 3 hours Maximum: 100 Marks
(including MCQs)

Draw neat diagrams where necessary. All questions are compulsory

3X10 30 marks

I. 32 years old lady married since 10 years presents with inability to conceive
(2+3+5=10)
a) Define infertility .
b) Enumerate the causes of female infertility.
c) What are the tests for ovulation?

II. 60 year oldpost menopausal woman complaints of irregular bleeding per vaginum of 15
days duration. Examination reveals a friable mass of 3cms limited to the cervix.

a) How will you evaluate this mass (1)


b) What is FIGO
staging for cancer cervix? (5)
c) what are the treatment modalities available for this patient (4)
III.

a)define and write minimum criteria to diagnose acute PID (3)

b) Enumerate the complications and sequelae of acute PID (4)

c) write the outpatient treatment in acute PID (3)

IV. Write short answers for the following(5X10=50)

1. What are the medico legal aspects of Medical Termination of Pregnancy? (5)
2. What are the non contraceptive advantages of the combined oral contraceptive pill?
(5)
3. A 55 years old lady who attained menopause 3 years ago presents with hot flushes,
irritability and cardiovascular complaints. What are the merits and demerits of
Hormone Replacement Therapy (HRT) in this patient? (5)
4. A 17 year old girl presents with history suggestive of primary amenorrhoea and on
general examination was found to be normal with normal sexual development. What
are the causes of cryptomenorhoea? What is the management of imperforate hymen

(2+3=5)
122

5. A 25years old Primipara had an unprotected sexual contact. What are the methods of
emergency contraception in this patient? (5)
6. A 22 years old woman presents with congestive dysmenorrhoea and dyspareunia. She
underwent laparoscopy and found to have moderate endometriosis. What is the
medical management of this condition? (5)

7. What are the clinical features trichomoniasis and how will you treat this condition in a
40 years old sexually active woman. (2+3=5)

8. A19 years old presents with lower abdominal pain of one month duration. Clinical
examination reveals a cystic mass in left adenexa and ultrasound shows a 8 cm cyst
with fat and doubtful calcified elements. What is your diagnosis and management?
(1+4=5)
9. Write the FIGO classification of abnormal uterine bleeding? (5)
10. What are all the genital tract injuries during child birth? How will you prevent
perineal tears? (2+3=5)
123

Practical examination Total marks -200

Subsections Marks allotted


OSPE/OSCE 60
CLINICAL EXAMINATION
Obs. Long Case 50
Gynaec.LongCase 50
ORAL EXAMINATION (Structured)
Obs 20
Gynae 20

The long caseshould be examined in a objective structured way under the following headings
on the basis of presentation of history, physical examination, appropriate investigations in a
logical sequence, appropriate management and clinical acumen.

Viva topics:ORAL EXAMINATION (Structured)

1. Family planning - 10 marks


2. Operative procedures - 10 marks GYN

3. AETCOM - 10 marks
4. Dummy and Pelvis - 10 marks OBS

OSPE / OSCE

There will be 8 static non observed stations, and 4 interactive/ observed stations. Out of the
total 12 stations 6 will be of Obstetrics and 6 will be of Gynecology. The time allotted to each
station will be 5 minutes.

DETAILED INSTRUCTIONS

1. The candidates must display their exam numbers on their white coats throughout the
examination.

2. Answer sheet: ● Prior to entering the examination hall each candidate will be provided
with aanswer sheet. Candidates must write their roll number on the response sheet before
starting examination. ● The candidate will only carry his/her response sheet while rotating
through the stations. No other papers will be allowed in the examination hall.
3. The candidate is not supposed to remove any document or material from any station.
4. Each station will carry equal weightage. Every station must be attempted.
5. A specified time will be allotted at each station which will be signaled by a bell.
6. Candidates are not allowed to bring mobile phones in the Examination Hall.
124

Conduct of Examination

1. The examination is in the form of a circuit. At the start the candidate would occupy the
station allocated to him / her according to their roll number, and will move to the next
station when the bell rings.

2. At these stations clear instructions would be written for performance of a task. The
candidate is expected to read the instructions and act accordingly.

On unobserved / static stations the candidate will be presented with a clinical case,
laboratory data, x-ray, ultrasound, CT scan, instrument, specimen etc. and will be asked
to give written responses to questions asked.
In the observed / interactive stations the candidate will have to perform a procedure for
example taking history, performing clinical examination, counseling, assembling an
instrument etc. One examiner will be present at each such station and will either rate the
performance of the candidate or ask questions testing the reasoning and problem-solving
skills.
3. The performance of each candidate will be assessed by the examiners on a pre determined
assessment form and the candidates will have to submit written responses to one-best /
short answer questions in the response sheet.

4. Candidates will rotate through the stations in this way till they have completed the circuit.
They will move only in one direction as displayed in the hall by arrow marks and will not
be allowed to go back to the previous station.

Resource list for OSCE stations:

1. Counseling
2. Scenario
3. Lab report
4. Instrument
5. Specimen
6. Partograph
7. USG picture
8. NST
9. Contraceptives
10. Dummies
11. X-rays
12. Operation
13. Antenatal card
14. Operative notes
15. Endoscopic findings
125

PEDIATRICS

Practical Exam – Total marks 100

One pediatrics case – 30 marks


One newborn case – 30marks
OSCE (5stations 4 marks each) - 20marks
Viva Voce - 20 marks
(4 stations which include X rays, Instruments, Nutrition, Drugs and Vaccine)

Theory Exam – Total marks 100

MCQ - 20 marks
Long answer Questions (10 marks x 3) - 30 marks
Short answer questions (5 marksx10) - 50 marks

Long answer questions one question from General Pediatrics (Growth &
Development, Nutrition, Immunization, Infectious diseases) and another one from
Neonatology.

Short answer questions should cover all Systemic Pediatrics, Pediatrics


Surgery, Genetics and one question from AETCOM Module.
126

Theory Model Question Paper – Total marks 100 (Duration 3 hrs)

Answer all questions


A. Long answer questions
1. One year old child was brought with the complaints of loose Stools, Vomiting of two day
duration.
(4+4+2=10)
a) How will you access severity of dehydration?
b) How will you manage based on the severity of dehydration?
c) Mention the common complications anticipated.

2. A healthy newborn on day one of life was brought with jaundice.


(3+2+3+2=10)
a) What are the likely causes?
b) How will you investigate this neonate?
c) What are the treatment options available?
d) What are the likely complications of delayed treatment?

3. A 2 year old child presented withhypothermia, poor peripheral pulses, generalsed


anasarca, withweight of 4kg with blood sugar level of 20mg/dl with severe undernutrton.
a) What is the diagnoses?
b) How will you classify severe undernutrition in children?
(1+2+3+3+2=10)
c) What are the investigationsyouwill do in thischild?
d) How will you manage this child?
e) What are likely complications?

B. Short Answer questions: (5x10=50)


4. Management of acute severe asthma.
5. Describe the clinical features and management of congestive cardiac failure in a one year
old child.
6. Complications of Nephrotic syndrome.
7. Counseling of a mother with Down syndrome baby.
8. Management of organo phosphorous poisoning.
9. Types and clinical presentation of 126rachea-esophageal fistula.
10. Clinical features and management of congenital hypothyroidism.
11. Prevention of mother to child transmission of HIV.
12. Laboratory Diagnosis and management of iron deficiency anaemia.
13. Enumerate the differences in the CSF findings between pyogenic and tubercular
meningitis.
-------

You might also like